Anda di halaman 1dari 97

See

discussions, stats, and author profiles for this publication at: https://www.researchgate.net/publication/308404215

Tutor senior olimpiade matematika lima benua


tingkat SMP

Book · January 2012

CITATIONS READS

0 1,342

1 author:

Nanang Susyanto
Gadjah Mada University
9 PUBLICATIONS 6 CITATIONS

SEE PROFILE

Some of the authors of this publication are also working on these related projects:

Biometric Fusion View project

All content following this page was uploaded by Nanang Susyanto on 06 October 2016.

The user has requested enhancement of the downloaded file.


Kata Pengantar

Olimpiade matematika bertujuan untuk melatih logika dan kemampuan menganalisa


suatu masalah matematika (problem solving). Materi yang dipelajari adalah matematika
dasar. Secara umum materi untuk olimpiade matematika tingkat SMP adalah teori
bilangan, aljabar, geometri, serta peluang dan statistika. Selain itu, ada juga kapita
selekta yang biasanya berisi penggunaan matematika dalam kehidupan sehari-hari,
kemampuan menyerap materi baru (definisi baru), serta soal tentang permainan. Tidak
ada teori khusus untuk kapita selekta ini, di sini kita dituntut untuk menggunakan
aljabar, teori bilangan, peluang dan statistika, serta geometri secara bersamaan. Namun
demikian, pembaca disarankan untuk memahami materi sekolah secara baik karena ini
akan menjadi dasar dari setiap materi pada buku ini.

Buku ini menyajikan review tentang materi olimpiade yanng telah dipelajari di sekolah
dan juga menawarkan definisi baru dari materi yang tidak didapatkan di sekolah serta
dilengkapi dengan contoh soal-soal dari OSK, OSP, dan OSN. Di samping itu, dalam
buku ini disediakan soal-soal OSK, OSP, dan OSN dua tahu terakhir yaitu 2010 dan
2011. Namun sayangnya untuk tahun 2010 penulis tidak berhasil menemukan soal OSN
2010 sehingga diganti dengan OSN 2009. Menariknya selain soal-soal dari olimpiade-
olimpiade di Indonesia, buku ini juga menyajikan soal olimpiade nasional dari beberapa
negara mulai dari yang mudah sampai yang sulit. Negara-negara yang dipilih penulis
adalah United Kingdom, United States of America, The Netherlands, Australia, dan
Malaysia. Diharapkan dari soal-soal dari luar Indonesia kita dapat melihat medan
”pertempuran” lomba-lomba internasional. Tidak lupa penulis juga memberikan soal
olimpiade internasional yaitu Junior Balkan. Jangan khawatir, karena semua soal-soal
dibahas pada bab tersendiri.

Meskipun semua soal dilengkapi dengan solusi, pembaca disarankan untuk mengerjakan
sendiri soal-soal yang ada. Ingat bahwa matematika adalah ilmu yang dipelajari dengan
cara cara berlatih learning by doing. Usahakan membaca solusi hanya jika kita Anda
sudah buntu dan tidak punya ide sama sekali.

Nanang S
Model soal olimpiade adalah soal-soal yang tidak rutin atau berorientasi pada
pemecahan masalah (problem solving). Ini berarti bahwa untuk menyelesaikan soal
olimpiade memerlukan kematangan matematika yaitu wawasan, kecermatan, ketelitian,
kejelian, kecerdikan, dan pengalaman. Hal-hal tersebut akan kita dapatkan dengan
memahami konsep matematika yang ada di sekolah secara baik dan benar serta berlatih
soal yang berbau problem solving yang tentu saja terdapat dalam buku ini.

Di dalam buku ini, dijelaskan beberapa trik untuk mengerjakan soal olimpiade. Ada
beberapa teknik dasar untuk menyelesaikan beberapa soal yang punya ciri khusus
diantaranya membaca pola, telescoping, pembatasan, membagi kasus, dan lain
sebagainya. Teknik membaca pola dapat kita temui pada saat kita membahas barisan
bilangan dan peluang, telescoping dapat kita temui saat kita membahas barisan
bilangan, pembatasan dapat kita temui saat membahas persamaan Diophantine dan
peluang, serta membagi kasus dapat kita temukan pada saat kita membahas peluang dan
persamaan. Jangan khawatir!!!!

Sebagai tambahan pengetahuan, dalam buku ini juga dibahas beberapa materi
pengayaan. Untuk sampai level nasional, materi pengayaan ini sangat jarang keluar
bahkah dapat dikatakan tidak keluar mulai dari OSK, OSP, maupun OSN. Namun
demikian, penulis menyajikan materi pengayaan ini karena dalam olimpiade nasional
SMP negara lain sudah sampai materi tersebut. Di samping itu, tidak ada ruginya kita
belajar materi pengayaan tersebut karena kelak di olimpiade SMA materi-materi
tersebut merupakan materi wajib.

Well, semoga buku ini bermanfaat dan kelak dapat membawa Anda untuk berprestasi
matematika di level internasional dan membawa nama harum bangsa kita tercinta....

Yogyakarta, Pebruari 2012

Penulis

Nanang S
Pola Seleksi Olimpiade Matematika SMP

Seleksi Olimpiade Matematika SMP dilaksanakan secara berjenjang mulai dari tingkat
sekolah, kabupaten/kota, propinsi, dan terakhir adalah Olimpiade Sains Nasional.
1. Seleksi tingkat sekolah dilaksanakan oleh masing-masing sekolah untuk
memilih wakil sekolah tersebut yang akan diikutkan ke seleksi tingkat
kabupaten/kota. Materi dan bentuk soal untuk tingkat ini diserahkan ke sekolah
masing-masing.
Peserta seleksi tingkat sekolah ini adalah siswa SMP/MTs kelas 1 dan 2 baik
negeri maupun swasta dengan nilai rapor Matematika minimal 7.
2. Seleksi tingkat kabupaten/kota dilaksanakan sekitar bulan Mei atau Juni. Bentuk
soalnya adalah tes tertulis 10 soal pilihan ganda dan 10 soal isian singkat. Materi
untuk seleksi ini adalah materi sekolah dan sedikit soal tentang pengenalan
problem solving.
Peserta Olimpiade tingkat kabupaten/kota adalah peserta perwakilan dari
sekolah. Setiap sekolah maksimal mengirimkan 5 wakil atau tergantung
kebijakan pemerintah daerah kabupaten.
3. Seleksi tingkat propinsi dilaksanakan sekitar bulan Juli. Bentuk soalnya adalah
tes tertulis 10 soal isian singkat dan 5 soal uraian (pernah juga 10 soal pilihan
ganda, 10 soal isian singkat, dan 2 soal uraian). Untuk seleksi ini, soal yang
berbau problem solving semakin banyak dan kental.
Peserta Olimpiade tingkat propinsi adalah peserta yang terpilih dari seleksi
tingkat kabupaten/kota. Biasanya, masing-masing kabupaten mengirimkan 3
wakilnya atau tergantung kebijakan pemerintan daerah propinsi.
4. Olimpiade Sains Nasional (Matematika) dilaksanakan sekitar bulan Agustus atau
September. Bentuk soalnya adalah tes tertulis 10 soal uraian. Tes dilaksanakan 2
hari, di mana setiap hari diujikan 5 soal dengan waktu 4 jam. Untuk tingkat
nasional ini, soal yang disajikan bau problem solving terasa sangat kental.
Peserta Olimpiade Sains Nasional adalah juara I pada seleksi tingkat propinsi
pada setiap propinsi ditambah dengan peserta yang terpilih melalui
perangkingan nasional hasil seleksi tingkat propinsi. Pada OSN ini diperebutkan
30 medali yaitu 5 emas, 10 perak, dan 15 perunggu.

Nanang S
Olimpiade Sains Nasional bukan akhir dari kompetisi olimpiade. Bagi peserta OSN
akan diikutkan seleksi calon peserta IJSO untuk menjadi wakil Indonesia, dan khusus
untuk peserta OSN Matematika akan diikutkan seleksi calon peserta IWYMIC yang
merupakan kompetisi matematika tingkat internasional.

Nanang S
Silabus Olimpiade Matematika SMP

Materi olimpiade matematika SMP adalah materi sekolah SMP dan pendalamannya
ditambah beberapa materi baru. Secara umum, ada 4 bidang yang diujikan dalam
olimpiade matematika yaitu Aljabar, Teori Bilangan, Geometri, serta Peluang dan
Statistika. Pokok bahasan yang diberi perhatian khusus, dapat dilihat pada penjelasan di
bawah ini.

Teori Bilangan
Sifat-sifat operasi pada himpunan bilangan bulat.
Pembagian Bersisa
Sifat-sifat operasi pada himpunan bilangan rasional
Sifat-sifat operasi pada himpunan bilangan real.
 Klasifikasi bilangan (bulat, pecahan, rasional, irrasional)
 Merasionalkan bentuk akar.
FPB dan KPK

Aljabar
Himpunan
 Himpunan bagian
 Operasi dua himpunan
Fungsi
 Pengertian fungsi
 Sifat-sifat fungsi secara umum
Perbandingan
 Perbandingan senilai
 Perbandingan berbalik nilai
Faktorisasi suku aljabar
 Bentuk a 2  b 2   a  b  a  b 
 Bentuk  a  b 
n

Persamaan garis lurus


Pertidaksamaan linier satu variabel
Sistem persamaan linier dua variabel
Eksponen dan logaritma
Pola bilangan
Persamaan kuadrat

Geometri
Bangun datar
 Segi-n dan lingkaran
 Garis tinggi dan garis berat segitiga
 Titik berat segitiga

Nanang S
Bangun ruang
 Volume tabung, kerucut dan bola
 Volume tabung dan kerucut terpancung
 Luas selimut tabung, kerucut dan bola
 Luas selimut tabung dan kerucut terpancung
Dalil Pythagoras
Trigonometri

Peluang dan Statistika


 Peluang kejadian
 Ukuran pemusatan

Kapita Selekta
Penggunaan matematika dalam kehidupan sehari-hari
Kemampuan menyerap materi baru (definisi baru)

Untuk seleksi tingkat kabupaten materi yang diujikan adalah semua materi di atas
namun dengan kedalaman yang dangkal yakni bau problem solving masih belum
sedikit, sedangkan untuk seleksi tingkat propinsi bau problem solving sedikit lebih
tajam yakni sebagian besar soal merupakan soal yang tidak rutin. Untuk tingkat
nasional, soal yang diujikan benar-benar soal yang tidak rutin, bau problem solving
benar-benar sangat kental dan untuk menyelesaikan soal dibutuhkan kreativitas dan
pengalaman.

Buku ini mengacu pada silabus OSN di atas. Anda juga akan mendapatkan pengalaman
yang sangat berarti dan akan sangat membantu Anda untuk mengetahui medan “perang”
olimpide matematika SMP sehingga kita tahu bagaimana cara “menggempur” soal-soal
tersebut.

Nanang S
Aljabar

Aljabar adalah materi dasar yang digunakan untuk memahami bidang-bidang lainnya.
Materi ini sebagian besar sudah dipelajari di sekolah (materi-materi rutin). Dengan
demikian, di sini kita hanya akan memperdalam pengetahuan dan memperlihatkan teknik
penyelesaian dalam soal nyata di olimpiade.

A. Persamaan dan Pertidaksamaan

Dalam memahami materi-materi aljabar, kita tidak bisa terlepas dari persamaan. Oleh
karena itu, diharapkan pembaca sudah menguasai persamaan yang telah dipelajari di
sekolah yaitu teknik-teknik menyelesaikan persamaan ataupun sistem persamaan seperti
metode grafik, metode subtitusi, dan metode eliminasi. Di sini kita akan mempelajari
teknik tidak rutin selain tiga teknik tersebut.

Contoh
Jika a + b = 1, b + c = 2, dan c + a = 3, maka a + b + c = .... (OSP 2004)
Jawaban:
Jumlahkan ketiga persamaan yang diketahui kita punya 2  a  b  c   1  2  3  6 ,

sehingga a  b  c  3. .

Contoh
Buktikan bahwa jika a  2 dan b  3 maka ab  6  3a  2b . (OSN 2003)
Jawaban:
Karena a  2 dan b  3 maka a  2  0 dan b  3  0 , sehingga
 a  2  b  3  0  ab  3a  2b  6  0  ab  6  3a  b.
Terbukti. .

B. Fungsi
Materi fungsi atau pemetaan tentunya telah akrab di telinga kita. Jika kita membahas
masalah fungsi tentunya kita tidak boleh hal-hal yang menyangkut fungsi antara lain
daerah asal (domain), daerah kawan (kodomain), daerah hasil (range), operasi-operasi di
antara fungsi-fungsi. Nah, khusus di olimpiade kita akan menitikberatkan pada sifat-sifat
fungsi.

Fungsi atau pemetaan f dari suatu himpunan A ke suatu himpunan B ditulis dengan
f : A  B didefinisikan sebagai relasi yang mengawankan setiap elemen di A dengan
tepat satu elemen di B. Selanjutnya A kita sebut dengan daerah asal atau domain,
sedangkan B kita sebut dengan daerah kawan atau kodomain. Jika di materi sekolah
biasanya kita diberikan rumus fungsinya kemudian kita diminta mencari sifat-sifatnya.
Nah, di olimpiade kita diberikan sifat-sifatnya kemudian kita diminta menentukan
rumusnya atau mencari sifat yang lainnya seperti mencari pembuat nol, mencari
maksimum atau minimum, ataupun mencari nilai di titik tertentu.

Contoh
Diberikan fungsi kuadrat f  x   ax 2  3x  c . Jika f 1  4 dan f  2   7 , maka

f  1  ... (OSP 2006)

Jawaban:
f 1  4  a  3  c  4  a  c  7 , sehingga f  1  a  3  c  7  3  10 . .

C. Faktorisasi Bentuk Aljabar

Pada materi kombinatorika kita akan membahas Binomial Newton atau bentuk  a  b  .
n

Pada sub bab ini kita akan konsenterasi pada faktorisasi/pemfaktoran x3  y3 dan

x n  y n , khususnya untuk n  2 dan n  3 . Faktorisasi ini sangat berguna dalam


menyelesaikan suatu persamaan maupun sistem persamaan, merasionalkan bentuk akar,
sampai menyederhanakan bentuk tertentu.
Dengan mudah dapat dicek bahwa x3  y 3   x  y   x 2  xy  y 2  dan untuk setiap

bilangan asli n kita punya x n  y n   x  y   x n 1  x n  2 y  ...  xy n  2  y n 1  , khusus untuk

n2 dan n3 berturut-turut kita punya x 2  y 2   x  y  x  y  dan

x3  y 3   x  y   x 2  xy  y 2  .

Contoh

 y  x  y  z
2 2

Jika   y  z , dan x  z maka nilai y sama sengan… (OSP 2009)


zx zx
Jawaban:

yz 
 y  x
2


 y  z
2


 y  x   y  z 
2 2


 y  x    y  z  y  x    y  z 
zx zx zx zx


 y  x  y  z  y  x  y  z    z  x  2 y  x  z   2 y  x  z.
zx zx
Dengan demikian, y   z  x  z  x . Jadi y  x .

Contoh
Semua bilangan real x yang memenuhi persamaan 3
x  4  3 x  1adalah… (OSP 2009)
Jawaban:
Misalkan 3
x  4  a dan 3
x  b , maka a  b  1 dan a3  b3  x  4  x  4 , sehingga

4  a 3  b3   a  b   a 2  ab  b 2   a 2  ab  b 2 . Perhatikan bahwa a  b  1, sebagai

akibatnya kita punya 4   b  1   b  1 b  b 2  b 2  2b  1  b 2  b  b 2  3b 2  3b  1


2

yang ekivalen dengan 3b2  3b  3  0  b2  b 1  0 . Dengan menggunakan rumus

solusi persamaan kuadrat (lihat sub bab D) solusi persamaan ini adalah b 
1
2
1  5 
1
  1
 
3
atau b 1 5 yang selanjutnya kita peroleh x  b3  1  5 atau
2 8
1
 
3
x  b3   1  5 . Jadi bilangan real x yang memenuhi persamaan 3
x  4  3 x 1
8
1
  1
 
3 3
adalah 1  5 dan  1 5 . .
8 8
D. Kuadrat sempurna dan Persamaan Kuadrat

Bentuk kuadrat sempurna merupakan bentuk yang sederhana tetapi sangat powerful
dalam menyelesaikan soal aljabar. Di sini kita akan mulai dengan 2 fakta sederhana:
i.  x  y   x 2  2 xy  y 2
2

ii. untuk setiap bilangan real x kita punya x2  0 , dan x2  0 jika dan hanya jika
x 0.
Poin (i) biasanya digunakan untuk memudahkan penghitungan tertentu dan
menyelesaikan persamaan kuadrat, sedangkan poin (ii) biasanya digunakan dalam
ketaksamaan dan mencari nilai maksimum/minimum.

Contoh
1 Ax 2 5
Diketahui bahwa x   7 . Tentukan A agar 4  . (OSN 2007)
x x  x 1 6
2

Jawaban:
2
1  1 1 1
Dari x   7 kita punya 7 2   x    x 2  2  2 , sehingga x 2  2  47. Dengan
x  x x x
demikian
Ax 2 5 5 x4  x2  1 5  2 1  5
  A     x  1  2    48   40.
x  x 1 6
4 2
6 x 2
6 x  6
Jadi A  40. .

Persamaan kuadrat adalah persamaan yang sederhana selain persamaan linear dengan
satu variabel. Bentuk umum dari persamaan kuadrat adalah ax2  bx  c  0 dengan
a, b, c  , dan a  0 . Solusi dari persamaan ini sering disebut sebagai akar persamaan
kuadrat. Ada beberapa cara menyelesaikan persamaan kuadrat:
i. dengan pemfaktoran yaitu dengan mengubah bentuk ax2  bx  c  0 menjadi
a  x  x1  x  x2   0 sehingga solusinya adalah x  x1 atau x  x2 .

ii. dengan menggunakan rumus persamaan kuadrat. Akan kita turunkan rumus
persamaan kuadrat dengan melengkapkan kuadrat sempurna.
b c
ax 2  bx  c  0  x2  x 0
a a
2 2
b  b   b  c
 x2  x    
a  2 a   2a  a
2
 b  b 2 4ac
  x    
 2a  4a 2 4a 2
b  b 2  4ac
2

  x   
 2a  4a 2
b b 2  4ac
 x 
2a 2a
b  b 2  4ac
 x .
2a

b  D
Rumus penyelesaian persamaan kuadrat di atas dapat ditulis x1,2  dengan
2a
D  b2  4ac yang biasanya disebut dengan diskriminan. Sekarang perhatikan bahwa

rumus di atas memuat bentuk D , dan kita tahu bahwa bilangan di dalam tanda akar
hanya akan terdefinisi jika tak negatif. Dari sini kita dapat turunkan dua teorema berikut.

Teorema
Persamaan kuadrat mempunyai persamaan/solusi real jika dan hanya jika diskriminannya
tak negatif.
Bukti:
Dapat dilihat langsung dari rumus penyelesaian persamaan kuadrat.

Teorema (jumlah dan hasil kali akar-akar persamaan kuadrat)


Jika x1 dan x2 adalah solusi-solusi dari persamaan kuadrat ax2  bx  c  0 maka

b c
x1  x2   dan x1 x2  .
a a
Bukti:
b  D
Dari rumus penyelesaian persamaan kuadrat kita punya x1  dan
2a

b  D
x2  . Dengan demikian, kita peroleh
2a

b  D b  D b  b 2b b
x1  x2       dan
2a 2a 2a 2a a

 b  D  b  D  b 2  D b   b  4ac  4ac c
2 2

x1 x2   
     2 .
 2a  2a  4a 2 4a 2 4a a

Contoh
Tentukan m agar persamaan  2 x 2  2mx   m  1  x 2  mx  1  0 mempunyai tepat dua

solusi real. (OSP 2006)


Jawaban:
Perhatikan persamaan kuadrat 2 x 2  2mx   m  1  0 . Diskriminan dari persamaan ini

adalah

D   2m   4  2     m  1   4m 2  8m  8  4  m 2  2m  2   4  m  1  4  4  0 ,
2 2

sehingga persamaan pasti mempunyai 2 solusi real berbeda. Dengan demikian agar
persamaan  2x 2
 2mx   m  1  x 2  mx  1  0 mempunyai tepat dua solusi real

berbeda, haruslah persamaan x2  mx  1 tidak punya solusi real atau dengan kata lain
D  0 yakni m2  4  0  2  m  2.
Jadi m yang dimaksud adalah 2  m  2. .

E. Ketaksamaan

Ketaksamaan yang akan kita bahas di sini adalah ketaksamaan yang berdasar pada fakta
no square is negative atau dengan kata lain ‘untuk setiap bilangan real x berlaku x2  0 ,
dan x2  0 jika dan hanya jika x  0 ’. Dari fakta yang sederhana ini kita dapat
menurunkan ketaksamaan AM-GM-HM.
Teorema
Untuk setiap bilangan-bilangan real positif x, dan y berlaku
x y 2
 xy  ,
2 1 1

x y
dan kesamaan terjadi jika dan hanya jika x  y .
Bukti:

 
2
Jelas bahwa x y  0 , sehingga

x y
x  2 xy  y  0  x  y  2 xy   xy .
2
2
Dengan mengalikam kedua ruas dengan pada ketaksamaan terakhir kita peroleh
xy

x  y 2 xy 1 1 2 2
     xy  .
xy xy x y xy 1 1

x y

 
2
Perhatikan bahwa kita menggunakan fakta x y  0 , sehingga kesamaan terjadi

x y 2
artinya  xy  jika dan hanya jika x  y .
2 1 1

x y

x y 2
Biasanya , xy , dan berturut-turut kita sebut dengan Arithmetic Mean
2 1 1

x y
(AM), Geometric Mean (GM), dan Harmonic Mean (HM), sehingga ketaksamaan
tersebut dapat kita tulis sebagai: AM  GM  HM . Hal yang perlu digarisbawahi bahwa
ketaksamaan AM  GM  HM hanya dapat digunakan untuk bilangan-bilangan real
positif. Jika tidak ada syarat demikian, maka kita kembali memakai fakta no square is
negative.

Salah satu kegunaan ketaksamaan adalah mencari nilai maksimum atau minimum dari
suatu fungsi atau ekspresi tertentu. Sebagai contoh jika kita telah dapat membuktikan
f  x   M untuk setiap x dan kita dapat menemukan nilai x sehingga f  x   M maka

kita katakana M adalah nilai maksimum dari fungsi f. Kebalikannya jika kita telah dapat
membuktikan f  x   m untuk setiap x dan kita dapat menemukan nilai x sehingga

f  x   m maka kita katakana m adalah nilai minimum dari fungsi f.

Contoh
Jumlah dua bilangan sama dengan 12. Hasil kali dua bilangan tersebut nilainya akan
paling besar jika salah satu bilangannya adalah ...... (OSK 2003)
Jawaban:
Misal salah satu bilangan tersebut adalah x , maka bilangan yang lain adalah 12  x .
Dengan demikian hasil kalinya adalah

x 12  x   12 x  x 2    x 2  12 x  36   36    x  6   36  36 . Jadi nilai maksimum


2

dari hasil perkaliannya adalah 36 dan terjadi saat x  6  0 atau dengan kata lain saat
x 6. .

Contoh
Diketahui a2  b2  1 dan x 2  y 2  1 . Lanjutkan proses berikut.

a 2
 b 2  x 2  y 2    ax  by   ...
2

a. Hubungan apakah yang bisa disimpulkan antara ax  by dengan 1?


b. Mengapa?
(OSN 2006)
Jawaban:

a 2
 b 2  x 2  y 2    ax  by   a 2 x 2  a 2 y 2  b 2 x 2  b 2 y 2  a 2 x 2  2abxy  b 2 y 2
2

 a 2 y 2  b 2 x 2  2abxy
  ay  bx  .
2

a. ax  by selalu kurang dari atau sama dengan 1.

b. Kita punya bahwa  ay  bx   0 sehingga


2
a 2
 b 2  x 2  y 2    ax  by   0  1.1   ax  by   0   ax  by   1
2 2 2
yang

tentu akan berakibat ax  by  1 . .

F. Perbandingan

Pada dasarnya perbandingan melihat hubungan antara dua variabel atau lebih. Ada dua
jenis perbandingan yaitu perbandingan senilai dan perbandingan berbalik nilai.
Perbandingan senilai adalah suatu perbandingan dengan sifat jika salah satu variabel naik
maka variabel yang lain juga ikut naik, dan juga sebaliknya jika salah satu variabel turun
maka variabel yang lain juga ikut turun. Sedangkan perbandingan berbalik nilai adalah
perbandingan dengan sifat jika salah satu variabel naik maka variabel yang lain justru
turun, begitu juga sebaliknya. Mari kita lihat contoh yang sangat sering kita jumpai dalam
kehidupan sehari-hari yaitu tentang waktu, jarak, dan kecepatan. Kita punya definisi
bahwa kecepatan rata-rata adalah jarak yang ditempuh setiap satuan waktu, atau secara
matematika dapat kita tuliskan sebagai:
s
v ,
t
Dengan v menyatakan kecepatan rata-rata, s menyatakan jarak/hasil, dan t menyatakan
waktu. Di dalam ilmu fisika, biasanya satuan dari s, t , dan v berturut-turut adalah
m, s (detik), dan m / s . Akan tetapi, sebenarnya v bukan hanya menyatakan kecepatan
rata-rata dalam satuan m / s melainkan juga kecepatan rata-rata menyelesaikan pekerjaan
seseorang, kecepatan mengisi air suatu kran, dan sebagainya.
s
Perhatikan kembali rumus v  . Jelas bahwa jika jarak naik(semakin besar) maka
t
kecepatan juga naik(semakin besar), begitu juga sebaliknya, sehingga hubungan antara
jarak dan kecepatan merupakan perbandingan senilai. Di lain pihak, jika waktu membesar
maka kecepatan mengecil(turun), demikian juga sebaliknya, sehingga hubungan antara
waktu dan kecepatan merupakan perbandingan berbalik nilai. Untuk lebih jelasnya mari
kita lihat beberapa contoh di soal olimpiade.
Contoh
Kendaraan A berjalan dengan laju 60 km/jam. Dua jam berikutnya kendaraan B
berjalan dengan laju 80 km/jam berangkat dari tempat dan menuju arah yang
sama. Setelah berapa jam kendaraan B menyusul kendaraan A?
a. 2 jam c. 4 jam e. 6 jam
b. 3 jam d. 5 jam
(OSK 2003)
Jawaban:
Misal B menyusul A setelah x . Waktu tempuh kendaraan A dan B berturut-turut adalah
x  2 dan x , sehingga jarak yang ditempuh kendaraan A dan B berturut-turut adalah
60  x  2  dan 80x . Kendaraan B menyusul A terjadi saat jarak yang ditempuh B sama

dengan A yaitu 60  x  2   80 x  3  x  2   4 x  x  6 . Jadi kendaraan B menyusul A

setelah 6 jam. (E).

Contoh
Tujuh ekor kambing menghabiskan rumput seluas 7 kali ukuran lapangan sepak bola
dalam waktu 7 hari. Waktu yang diperlukan oleh 3 ekor kambing untuk menghabiskan
rumput seluas 3 kali ukuran lapangan sepak bola adalah ..hari. (OSK 2004)
Jawaban:
Kita punya bahwa kecepatan makan 7 ekor kambing adalah
7 lapangan
 1 lapangan/hari, sehingga kecepatan makan 1 ekor kambing
7 hari
1 3
lapangan/hari, akibatnya kecepatan makan 3 ekor kambing lapangan/hari. Dengan
7 7
3
demikian, rumput seluas 3 lapangan akan dihabiskan dalam waktu  7 hari. .
3/7

G. Eksponen

Dalam istilah Indonesia, eksponen lebih dikenal dengan nama pangkat. Di sini kita akan
membahas definisi, sifat, serta persamaan dan pertidaksamaan eksponen.
Perhatikan kesamaan ab  c . Bilangan a disebut bilangan pokok, b disebut
pangkat/eksponen, sedangkan c disebut hasil. Pada awalnya eksponen hanya
didefinisikan jika pangkatnya merupakan bilangan bulat. Namun dalam perkembangan
selanjutnya, definisi ini dapat diperluas jika pangkatnya merupakan bilangan rasional
bahkan bilangan real.

Definisi
Untuk sebarang bilangan real a dan bilangan bulat tak negatif n didefinisikan
1 jika n  0 dan a  0

a n  a  a  ...  a jika n  0.
 sebanyak n

Bentuk 00 merupakan bentuk tak tentu, sehingga kita tidak dapat mendefinisikan secara
tunggal nilai dari 00 . Selanjutnya, untuk n bilangan bulat negatif dan a bilangan real
1
tidak nol didefinisikan a n  . Sedangkan untuk sebarang bilangan bulat tak negatif n
an
1
dan bilangan real positif a , didefinisikan a n sebagai bilangan real positif x dengan sifat

x n  a yang selanjutnya ditulis dengan x  n a . Dari sini tentu saja kit adapt
m
mendefinisikan a n  n a m .

Contoh

b
Jika a  maka b dinyatakan dalam a adalah…
1 b
a2 a2
a. b  1  a 2 c. b  e. b 
1  a2 1  a2
1  a2 1  a2
b. b  d. b 
a2 a2
(OSK 2004)
Jawaban:
b b
Menurut definisi akar, kita punya a   a2  , sehingga
1 b 1 b
b  1  b  a 2  b  a 2  ba 2  b  ba 2  a 2  b 1  a 2   a 2 .

a2
Dengan demikian b  . (C).
1  a2

Dari definisi-definisi di atas, kita punya beberapa sifat sebagai berikut:


1. am  a n  a mn
2. am : a n  a mn

3.  a m   a mn
n

3.  ab   a nb n
n

Hal yang perlu diingat bahwa sifat 1 dan 2 tidak boleh dibalik yakni pada umumnya
am  an  amn dan juga am  a n  a mn .

Contoh
44  44  44  44  ...
a. 27 c. 1034 e. 512
b. 210 d. 54
(OSK 2003)
Jawaban:

44  44  44  44  4  44  41 4  45   22   210.(B). .
5

f  x
Persamaan eksponen yang akan kita bahas di sini adalah yang berbentuk a  a g  x

 
2
atau p a    qa f  x   r  0 .
f x

f  x
 Untuk persamaan tipe a  a g  x (tentu saja yang kita maksud adalah a  1 ,
f  x
karena jika a  1 maka tentu 1  1  a g  x berapapun x) penyelesaiannya sama
dengan penyelesaian persamaan f  x   g  x  . Jadi, langkah pertama yang kita

lakukan untuk menyelesaikan persamaan dengan bentuk tersebut adalah


menyamakan bilangan pokoknya, baru kemudian kita samakan pangkatnya.

 
2
 Persamaan eksponen tipe p a    qa f  x   r  0 . Untuk persamaan eksponen
f x

bentuk ini, kita dapat mengubah menjadi persamaan kuadrat dengan substitusi
y  a f  x  . Setelah dicari penyelesaian dari persamaan kuadrat yang kita peroleh
baru kemudian mencari x. Yang perlu untuk diingat adalah fakta yang
f  x
mengatakan bahwa jika a  0 , maka a  0.

Contoh
Jika sistem persamaan x 7 y 5  r dan x 4 y 3  s dengan x, y, r , dan s adalah bilangan

positif mempunyai penyelesaian x  r a sb dan y  r c s d , maka hasil dari a  b  c  d


adalah…
a. 19 c. 1 e. ½
b.2 d. 7/12
(OSP 2007)
Jawaban:

x 7 y 5  r   r a s b   r c s d   r  r 7 a 5c s 7 b 5 d  r artinya 7a  5c  1 dan 7b  5d  0


7 5

x 4 y 3  s   r a s b   r c s d   s  r 4 a 3c s 4b 3d  s artinya 4a  3c  0 dan 4b  3d  1 .


4 3

Dari sini kita punya dua sistem persamaan


7a  5c  1 7b  5d  0
 dan 
4a  3c  0 4b  3d  1.
4
Dari sistem persamaan yang pertama kita peroleh c a sehingga
3
20 1
7a  a  1  a  1  a  3. Sedangkan dari sistem yang kedua kita peroleh
3 3
7 21 1
d   b sehingga 4b  b  1   b  b  5.
5 5 5
4 7 1 2
Dengan demikian a  b  c  d  a  b  a  b   a  b  1  2  1. (C). .
3 5 3 5

Selain persamaan eksponen, kita juga mengenal pertidaksamaan eksponen. Sama


f  x
dengan persamaan eksponen, di sini kita akan membahas bentuk a  a g  x (dapat juga

 
2
, ,  ), dan bentuk p a    qa f  x   r  0(, , ) .
f x

 Tipe a f  x  a g  x : Perhatikan bahwa jika a  1 maka akan selalu terjadi


kesamaan, karena 1 pangkat berapapun selalu 1. Oleh sebab itu, kita asumsikan
a  1 . Untuk menyelesaikan pertidaksamaan bentuk tersebut, kita akan membagi
2 kasus sebagai berikut:
 Kasus a  1
Untuk a  1 , berlaku a    a    f  x   g  x  . Begitu juga jika tandanya
f x g x

diganti dengan , , atau  .


 Kasus a  1

Untuk kasus ini, berlaku a    a    f  x   g  x  . Begitu juga jika


f x g x

tandanya diganti dengan , , atau  .

 
2
 Tipe p a f  x  qa f  x   r  0 : Seperti halnya persamaan eksponen, cara

menyelesaian pertidaksamaan bentuk ini akan kita gunakan substitusi y  a f  x  .

Sehingga pertidaksamaan tersebut akan menjadi py 2  qy  r  0 , yang tentu saja


merupakan pertidaksamaan kuadrat, kemudian kita selesaikan lagi dengan
pertidaksamaan yang telah kita bahas sebelumnya.

H. Barisan dan Deret Bilangan

Suatu susunan bilangan terkadang dapat kita baca polanya, yakni secara logika kita akan
mengetahui bilangan yang akan muncul berikutnya jika kita diberikan suatu urutan
bilangan-bilangan. Bilangan-bilangan yang disusun/diurutkan menurut aturan tertentu
disebut dengan barisan bilangan. Sedangkan jumlahan dari barisan bilangan disebut
deret.

Secara matematis, barisan bilangan adalah fungsi dengan domain himpunan bilangan
asli. Dan deret adalah jumlahan dari barisan barisan.

Barisan bilangan ini biasa dituliskan dengan u1 , u2 ,..., un . Sedangkan deret dituliskan

dengan sn  u1  u2  ..., un .

u1 disebut suku pertama, u 2 disebut suku kedua, ..., dan seterusnya secara umum u n

disebut suku ke-n.

Contoh
Pola ABBCCCDDDDABBCCCDDDDABBCCCDDDD.... berulang sampai tak
terhingga. Huruf apakah yang menempati urutan ke 2533 ? (OSN 2003)
Jawaban:
Perhatikan bahwa pola di atas berulang setiap 10 suku yaitu ABBCCCDDDD. Dengan
demikian, kita cukup mencari digit terakhir dari 2533 . Digit terakhir dari 2533  32  27
adalah 4, sehingga huruf yang menempati urutan ke 2533 sama dengan huruf keempat
dari barisan tersebut yaitu huruf C. .

Barisan aritmetika adalah barisan yang setiap sukunya diperoleh dengan menambah
suatu bilangan yang tetap (konstan) dari suku sebelumnya, penambah ini selanjutnya
disebut beda. Secara umum, barisan aritmetika berbentuk:
a, a  b, a  2b, a  3b,... dengan a adalah suku pertama dan b adalah beda.

Secara matematis, suku ke-n barisan aritmetika dapat dinyatakan secara ekplisit sebagai
un  a   n  1 b , dengan a dan b berturut-turut menyatakan suku pertama dan beda.

Bagaimana dengan jumlah suku-suku barisan aritmetika yang kita sebut dengan deret
aritmetika. Deret ke-n atau dengan kata lain jumlahan n suku pertama barisan aritmetika
kita tulis dengan S n . Perhatikan kembali bahwa Sn  u1  u2  ...  un . Jika kita balik

susunannya menjadi Sn  un  un1  ...  u1 , sehingga

2 S n  u1  u2  ...  un  un  un 1  ...  u1
  u1  un    u2  un 1   ...   un  u1 
  a  a   n  1 b    a  b  a   n  2  b   ...   a   n  1 b  a 
  2a   n  1 b    2a   n  1 b   ...   2a   n  1 b 
 n  2a   n  1 b  .

n  2a   n  1 b  .
1
Dengan demikian, kita punya Sn 
2
1
Karena un  a   n  1 b , maka S n juga dapat ditulis sebagai Sn  n  a  un  .
2
Jadi rumus deret aritmetika adalah:

n  2a   n  1 b 
1
Sn 
2
1
 n  a  un  .
2

Nah, sekarang bagaimana cara mencari u n jika diketahui rumus S n ? Mudah saja, karena

Sn  S n 1  u1  u2  ...  un 1  un   u1  u 2  ...  u n 1   u n , maka un  Sn  Sn1 untuk

n  2 dan tentu saja u1  S1 .

Contoh
Nilai dari 20092  20082  20072  20062  20052  ...  32  22  12 adalah… (OSP 2009)
Jawaban:
Kita akan menggunakan pemfaktoran x 2  y 2   x  y  x  y  .

S  20092  20082  2007 2  20062  20052  ...  32  2 2  12


  2009  2008  2009  2008    2007  2006  2007  2006   ...  3  2 3  2   1
 4017  4013  ...  5  1
 1  5  9  ...  4013  4017.
Perhatikan bahwa 1  5  9  ...  4013  4017 merupakan deret aritmetik dengan suku
awal 1, beda 4, dan suku terakhir 4017. Rumus umum dari barisan aritmetika
penyusunnya adalah un  a   n  1 b  1   n  1 4  4n  3 , sehingga karena

4n  3  4017 terpenenuhi saat n  1005 , maka banyak suku pada deret tersebut adalah
1005. Dengan demikian
1 1005 1  4017  1005  4018
S  n  a  un     1005  2009
2 2 2
 1005  2000  9   2.010.000  9045  2.019.045.

Jadi jumlahnya adalah 2.019.045. .

Barisan geometri adalah barisan yang setiap sukunya diperoleh dengan mengalikan
suatu bilangan yang tetap (konstan) dari suku sebelumnya. Pengali ini selanjutnya disebut
dengan rasio atau pengali. Dengan demikian barisan geometri adalah barisan yang setiap
sukunya diperoleh dengan mengalikan suatu bilangan tertentu dari suku sebelumnya.
Secara umum, barisan geometri berbentuk:
a, ar , ar 2 , ar 3 ,... dengan a adalah suku pertama dan r adalah rasio.

Secara matematis, rumus suku ke-n dari barisan geometri adalah un  ar n 1 , dengan a
adalah suku pertama dan r adalah rasio.

Selanjutnya, akan dibahas jumlahan dari suku-suku barisan geometri yang selnjutnya kita
sebut dengan deret geometri. Perhatikan kembali bahwa
S n  u1  u2  ...  un  a  ar  ...  ar n 1 .

Dengan mengalikan kedua ruas dengan r kita peroleh rS n  ar  ar 2  ...  ar n , sehingga

rS n  S n  ar  ar 2  ...  ar n   a  ar  ...  ar n 1 
  ar  ar 2  ...  ar n 1    ar  ar 2  ...  ar n 1   ar n  a
 ar n  a.

Oleh karena itu kita peroleh  r  1 Sn  a  r n  1  S n  a


r n
 1
.
 r  1
Perhatikan bahwa
r n
 1

1  r  , sehingga rumus di atas juga dapat ditulis dalam
n

 r  1 1  r 

bentuk lain yaitu: Sn a


1  r  . n

1  r 

Jadi Sn a
r n
 1
atau Sn a
1  r  . Bentuk pertama biasanya digunakan jika
n

r 1
 r  1 1  r 
, sedang bentuk yang kedua digunakan jika r  1 .

Seperti halnya pada deret aritmetika, pada deret geometripun juga berlaku un  Sn  Sn1

untuk n  2 dan tentu saja u1  S1 .

Contoh
Suatu barisan bilangan real mempunyai suku-suku didefinisikan sebagai
berikut.
un  ar n 1 jika n  4m  3 atau n  4m  2
un  ar n 1 jika n  4m 1 atau n  4m
dengan a  0, r  0 , dan m bilangan bulat positif.
Buktikan bahwa jumlah semua suku ke-1 sampai dengan suku ke-2009 adalah
a 1  r  r 2009  r 2010 
. (OSN 2008).
1 r2
Jawaban:
Beberapa suku dari barisan ini adalah a, ar, ar 2 , ar 3 , ar 4 , ar 5 , ar 6 , ar 7 ,... Barisan ini
hampir sama dengan barisan geometri hanya saja dia akan bertanda + + - - untuk suku ke
1,2,3,4 dan seterusnya dengan mengambil 4 suku berurutan. Misalkan
S  u1  u2  ...  u2008 dan T  u1  u2  ...  u2008  u2009  S  u2009  S  ar 2008 maka

S  a  ar  ar 2  ar 3  ...  ar 2004  ar 2005  ar 2006  ar 2007 . Dengan demikian


S  a  ar  ar 2  ar 3  ...  ar 2004  ar 2005  ar 2006  ar 2007
 a  ar  ar 2  ar 3  ...  ar 2004  ar 2005  ar 2006  ar 2007  2  ar 2  ar 3  ar 6  ar 7  ...  ar 2006  ar 2007 

1  r   2
2008

a
1 r
 ar 2
 ar 6  ...  ar 2006  ar 3  ar 7  ...  ar 2007 

1  r 2008 
a
1 r

 2a  r 2  r 6  ...  r 2006    r 3  r 7  ...  r 2007 
1  r 2008 
a
1 r

 2a  r 2  r 6  ...  r 2006   r  r 2  r 6  ...  r 2006  
a
1  r 2008   2a 1  r   r 2  r 6  ...  r 2006  .
1 r

Perhatikan bahwa r 2  r 6  ...  r 2006 merupakan deret geometri dengan suku awal r 2 dan
rasio r 4 . Perhatikan juga bahwa pangkat dari r merupakan barisan aritmetika dengan
suku awal 2 dan beda 4, sehingga rumus umumnya adalah un  2   n  1 4  4n  2 .

Karena 2006  4n  2 terpenuhi saat n  502 maka kita dapat simpulkan bahwa banyak
suku deret r 2  r 6  ...  r 2006 adalah 502. Dengan demikian

 1   r 4 502 
  r 2  1  r 
2008
r  r  ...  r
2 6 2006
r  2
4 
, sehingga
 1 r 4
  1 r 
 
 1  r 2008  2  1 r
2008

S  a   2 a 1  r  r  4 
 1 r   1 r 
 1 2 1  r  r 2 
 a 1  r 2008    
 1  r 1  r 2 1  r 2  
 
 1 2 1  r  r 2 
 a 1  r  
2008
 
 1  r 1  r 1  r  1  r 2  
 
a 1  r 2008   2r 2 
  1  2 
1 r  1 r 
a 1  r 2008   1  r 2  2r 2 
  
1 r  1 r
2

a 1  r 2008   1  r 2 
  2 
1 r  1 r 
a 1  r 2008   1  r 1  r  
  
1 r  1 r2 
 1 r 
 a 1  r 2008   2 
 1 r 
a 1  r  r 2008  r 2009 
 .
1 r2
Selanjutnya kita peroleh
T  S  ar 2008
a 1  r  r 2008  r 2009 
  ar 2008
1 r 2


a
1 r 2  
1  r  r 2008  r 2009   1  r 2  r 2008 
2 
1  r  r 2008  r 2009  r 2008  r 2010 
a

1 r

2 
1  r  r 2009  r 2010 
a

1 r
a 1  r  r 2009  r 2010 
 .
1 r2
Terbukti seperti yang diinginkan. .
Selain deret aritmetika dan deret geometri di atas, terdapat beberapa deret yang sebaiknya
kita mengetahuinya.
1. Deret bilangan asli: ini merupakan jumlah dari beberapa bilangan asli yang
1
pertama yaitu 1  2  3  ...  n  n  n  1 . (deret ini dapat dihitung dengan
2
menggunakan rumus deret aritmetika).
2. Deret bilangan kuadrat: ini merupakan jumlah dari beberapa kuadrat bilangan
1
asli yang pertama yaitu 12  22  32  ...  n 2  n  n  1 2n  1 .
6
3. Deret bilangan kubik (pangkat 3): ini merupakan jumlah dari beberapa pangkat
2
1 
tiga dari bilangan asli yang pertama yaitu 1  2  3  ...  n   n  n  1  .
3 3 3 3

2 

Dalam soal olimpiade, sering sekali kita menghitung deret bahkan perkalian dari
beberapa suku dari suatu barisan yang bukan aritmetika maupun geometri. Di sini kita
akan membahas teknik telescoping. Cara ini adadapat dilakukan untuk penjumlahan dan
perkalian.
1. Tipe penjumlahan u1  u2  ...  un . Jika kita dapat mengubah menjadi u1  x1  x0 ,

u2  x2  x1 , u3  x3  x2 ,…, un  xn  xn 1 , maka dengan mudah kita dapat

menghitung u1  u2  ...  un   x1  x0    x2  x1   ...   xn  xn 1   xn  x0 . Atau

dengan cara lain yakni dengan membalik urutan suku-sukunya seperti pada saat
kita membuktikan rumus deret aritmetika, yaitu jika kita punya
1
u1  un  u2  un1  ...  un  u1  c maka tentu saja u1  u2  ...  un  nc .
2
x1 x
2. Tipe perkalian u1u2 ...un . Jika kita dapat mengubah menjadi u1  , u2  2 ,
x0 x1
x3 x
u3  ,…, un  n , maka dengan mudah kita dapat menghitung
x2 xn 1

x1 x2 xn x
u1u2 ...un  . ...  n.
x0 x1 xn 1 x0
Contoh
1 1 1 1 1
Bentuk sederhana dari     ...  adalah… (OSK 2005)
2 6 12 20 2005  2005  1

Jawaban:
1 1 1 1 1 1 1 1 1 1 1
    ...       ...  
2 6 12 20 2005  2005  1 1.2 2.3 3.4 4.5 2004.2005 2005.2006
1 1 1 1 1 1 1
 1     ...    
2 2 3 2004 2005 2005 2006
1
 1
2006
2005
 .
2006

Contoh
x
Diketahui fungsi bilangan real f  x   , untuk x  1 . Nilai dari
1 x
1 1  1 
f  2007   f  2006   ...  f  3  f  2   f    f    ...  f 
2 3  2007 
adalah... (OSP 2007)
Jawaban:
1 1  1 
Misal S  f  2007   f  2006   ...  f  3  f  2   f    f    ...  f  .
2 3  2007 
Sekarang perhatikan bahwa
1 a 1/ a a 1 a 1 a 1
f a  f           1 untuk setiap a
 a  1  a 1  1/ a 1  a a  1 1  a 1  a 1  a
yang tidak nol, sehingga
1 1  1 
S  f  2007   f  2006   ...  f  3  f  2   f    f    ...  f  
2 3  2007 
  1    1    1 
  f  2   f      f  3  f     ...   f  2007   f  
  2    3    2007  
 2006  1
 2006.
Jadi jumla yang dimaksud adalah -2006. .
Contoh
Jika n adalah bilangan asli. Maka bentuk paling sederhana dari perkalian
 1  1  1   1 
1  2 1  2 1  2  ... 1  2  adalah… (OSP 2006)
 2  3  4   n 
Jawaban:
 1  1  1  1   1  1  1  1   1  1 
1  2 1  2 1  2  ... 1  2   1  1  1   1   ...  1   1  
 2  3  4   n   2  2  3  3   n  n 
 1  1   1  1  1   1 
 1  1   ... 1  1  1   ... 1  
 2  3   n  2  3   n 
 1 2 3 n  2 n  1  3 4 5 n n 1 
  . . ....  . . ... 
 2 3 4 n  1 n  2 3 4 n  1 n 
1 n 1
 .
n 2
n 1
 .
2n
n 1
Jadi bentuk sederhanya adalah . .
2n
Geometri

Geometri adalah cabang ilmu matematika yang mempelajari tentang ilmu ukur. Pada bab ini
kita akan mempelajari geometri bidang dan geometri ruang. Geometri bidang meliputi
pembahasan tentang segitiga, segi-n, kesebangunan, Teorema Pythagoras, lingkaran, serta
hubungan antara lingkaran dan segitiga. Sedangkan geometri ruang meliputi kubus, balok,
tabung, limas, kerucut, dan bola serta beberapa bangun terpancung. Tentu saja kita tidak akan
mulai semuanya dari awal, melainkan hanya akan mendalami ataupun memperkenalkan
sesuatu yang baru.

A. Segitiga

Segitiga adalah bangun datar bersisi lurus yang paling sederhana. Bahkan, semua bangun
datar yang bersisi lurus pasti dapat kita partisi menjadi beberapa segitiga.

Beberapa sifat dasar dan rumus rumus pada segitiga:


1. A  B  C  1800 ,
2. a  b  c , b  c  a , dan c  a  b (disebut ketaksamaan segitiga),
abc
3. Keliling  a  b  c dan didefinisikan s sebagai setengah keliling yakni s  ,
2
4. Luas segitiga ABC selanjutnya ditulis dengan  ABC  dapat dihitung dengan rumus

 panjang alas    tinggi 


 ABC  
2

5. Formula Heron:  ABC   s  s  a  s  b  s  c  dengan s menyatakan setengah

keliling.
Formula Heron merupakan cara alternatif untuk menghitung luas segitiga tanpa harus
menghitung tingginya terlebih dahulu. Perhatikan rumus luas pada poin (4). Jika rumus ini
a  ha
kita aplikasikan pada segitiga ABC di atas maka  ABC   dengan ha menyatakan
2
panjang garis tinggi yang ditarik dari titik sudut A. Sekarang perhatikan bahwa segitiga ABD
1
dan ADC mempunyai tinggi yang sama yaitu ha , sehingga
 ABD   2  BD  ha 
BD
. Dari
 ADC  1  DC  ha DC
2
sini kita menemukan fakta yang sederhana namun powerful dalam menyelesaikan soal yaitu
bahwa jika dua segitiga mempunyai tinggi yang sama maka perbandingan luasnya sama
dengan perbandingan panjang alasnya. Begitu juga sebaliknya, yaitu jika dua segitiga
mempunyai alas sama maka perbandingan luasnya sama dengan perbandingan tingginya.

Contoh
Gambar bangun berikut, ABCD adalah persegi dengan sisi 6 satuan. Titik E dan F membagi
diagonal AC menjadi tiga bagian sama panjang. Luas segitiga DEF = ....

(OSK 2003)
Jawaban:
Kita tarik garis DE dan DF.

Persegi ABCD terbagi menjadi 6 segitiga yang tingginya adalah setengah panjang diagonal
BD. Karena panjang semua alasnya sama maka keenam segitiga tersebut mempunyai luas
1 1
yang sama, sehingga  DEF    ABCD   .6.6  6 satuan luas. Jadi luas segitiga DEF
6 6
adalah 6 satuan luas. .

Contoh
Perhatikan Gambar 2, yaitu 4 buah layang-layang kongruen yang memuat pada persegi dan
ternyata masih tersisa daerah persegi yang diarsir. Jika panjang p  3 2 cm, dan q  5 2
cm, maka luas daerah yang diarsir adalah ….

(OSP 2009)
Jawaban:

 p  q    8 2 
2 2
 3 2 5 2  128 . Perhatikan bahwa setiap
2
Luas persegi yang besar

belah ketupat merupakan gabungan dari dua segitiga siku-siku dengan sisi-sisi siku-siku p
dan q. Karena ada 4 belah ketupat maka luas yang tidak diarsir sama dengan 8 kali luas
1
  
segitiga siku-siku tersebut yaitu  8. pq  4 3 2 5 2  120 . Jadi luas yang diarsir
2
adalah L  128 120  8 cm2. .
Telah dikatakan di atas bahwa sebarang segi-n dapat kita partisi menjadi segitiga-segitiga.
Mudah dipahami bahwa setiap segi-n dapat dibagi/dipartisi menjadi n  2 segitiga segitiga
yang semua titik sudutnya merupakan titik sudut segi-n.

Dengan demikian jumlah semua sudut pada segi-n adalah  n  2 1800 . Dari sini tentu kita

dapatkan besar setiap sudut segi-n beraturan adalah


 n  2  1800 .
n

Contoh
Tentukan bilangan asli n terkecil sehingga jumlah ukuran sudut-sudut segi-n, dengan n  6
kurang dari n 2 derajad. (OSN 2004)
Jawaban:
Kita tahu bahwa jumlah ukuran sudut-sudut segi-n adalah  n  2 1800 , sehingga kita akan

mencari bilangan asli terkecil n sehingga  n  2 180  n 2 yang ekivalen dengan

180n  n 2  360  n 180  n   360 . Perhatikan bahwa fungsi grafik f dengan rumus

f  x   180 x  x 2 merupakan parabola terbuka ke atas dengan persamaan sumbu simetri

x  90 , sehingga fungsi f merupakan fungsi naik untuk x  90 dan turun jika x  90 .


Karena f  3  3 180  3   360 maka agar n 180  n   360 haruslah x  90 (karena setelah

90 fungsi akan turun). Dengan demikian, untuk mencari n yang dimaksud kita cukup mencari
bilangan asli n sehingga  n  1 180   n  1   360 tetapi n 180  n   360 . Perhatikan

bahwa 177 180  177   531  360 dan 178 180  178   356  360 , sehingga n terkecil yang

dimaksud adalah n  178 . .


B. Kesebangunan

Mudahnya kata sebangun sama dengan serupa, artinya dua bangun datar dikatakan sebangun
jika bentuknya serupa meskipun ukurannya tidak harus sama. Secara formal, dua bangun
datar bersisi lurus dikatakan sebangun jika besar semua sudutnya sama dan perbandingan
panjang sisi-sisi yang bersesuaian sama. Sebagai contoh segilima ABCDE sebagun dengan
segiempat PQRST ditulis dengan ABCDE PQRST jika
AB BC CD DE EA
A  P, B  Q, C  R, D  S , E  T dan     k
PQ QR RS ST TP
untuk suatu konstanta k. Dalam hal k  1 , dikatakan bahwa dua bangun tersebut kongruen.

Hal yang istimewa dari dua bangun yang sebangun adalah kita dapat mencari sifat satu
bangun jika diketahui sifat bangun yang lainnya. Sebagai contoh misalkan diketahui dua
bangun datar sebangun katakan dengan perbandingan k, maka
1. Perbandingan kelilingnya adalah k,
2. Perbandingan luasnya adalah k 2 ,
3. Perbandingan panjang garis (baik diagonal, garis tinggi, atau apapun) yang bersesuaian
adalah k .

Salah satu penggunaan nyata dari kesebangunan adalah skala pada peta. Jelas bahwa setiap
peta sebangun dengan tempat aslinya. Misalkan perbandingan ukuran sebenarnya dan ukuran
pada peta adalah k maka skala peta tersebut adalah 1: k.
Contoh
Pada sebuah peta dengan skala 1 : 100.000, luas tanah sebuah sekolah adalah 50 cm2. Luas
tanah sekolah tersebut pada peta dengan skala 1 : 200.000 adalah ...... (OSK 2003)
Jawaban:
Jelas bahwa 2 peta tersebut sebangun dengan peta pertama lebih besar dari peta kedua.
Mudah dipahami bahwa perbandingan kesebangunan peta pertama dan peta kedua adalah 2,
50
sehingga luas tanah pada peta kecil adalah 2
 12,5 cm2. .
2

Syarat kesebangunan pada segitiga lebih sederhana daripada bangun segi-n yang lainnya.

Dua segitiga dikatakan sebangun jika memenuhi salah satu dari:


1. Ketiga sudutnya sama besar.
Ini juga dapat disederhanakan lagi karena jumlah besar sudut setiap segitiga adalah 1800
maka dua segitiga sebangun jika besar dua sudut yang bersesuaian sama.
A  P
AB AC BC
Jika B  Q maka ABC PQR akibatnya   .
PQ PR QR
C  R
2. Perbandingan ketiga sisi yang bersesuaian sama
AB AC BC
Jika   maka ABC PQR akibatnya A  P, B  Q, dan
PQ PR QR
C  R .
3. Perbandingan dua sisi yang bersesuaian sama dan sudut yang mengapit keduanya
besarnya sama
AB AC
Jika diketahui  dan A  P maka ini juga akan berakibat ABC PQR .
PQ PR
Perhatikan segitiga

 Jika DE // AB (sejajar) maka mudah dipahami bahwa CAB CDE yang berakibat
CD CE DE CD CE
  yang selanjutnya diperoleh juga  .
CA CB AB DA EB
CD CE
 Kebalikannya jika  maka CAB CDE sehingga DE // AB .
DA EB

Contoh
Perhatikan gambar berikut ini.

Dari gambar di atas diketahui bahwa jari- jari lingkaran kecil adalah 3 cm dan jari-jari lingkaran
besar adalah 5 cm. Panjang CD adalah ……cm. (OSP 2006)
Jawaban:
Tarik garis tegak lurus dari A dan B ke garis singgung singgung lingkaran.

DB PB DC  3 3
Karena PB // QA maka    . Dari sini kita peroleh 5DC  15  3DC  33
DA QA DC  11 5
yang selanjutnya diperoleh DC  9 cm. .

Sekarang perhatikan segitiga siku-siku berikut ini.

Dapat dibuktikan bahwa DAB DCA ACB , sehingga kita peroleh AC 2  CD  CB ,


AB2  BD  BC , dan AD2  DB  DC (ini disebut rumus proyeksi).

Selanjutnya kita juga punya


AB 2  AC 2  BD  BC  CD  BC   BD  DC   BC  BC  BC  BC 2 . Nah, rumus

AB2  AC 2  BC 2 ini yang disebut dengan Teorema Pythagoras.


Contoh
Perhatikan gambar berikut. Panjang CP adalah .....

(OSK 2004)
Jawaban:
Misal Q, R, S, dan T berturut-turut adalah proyeksi P pada AB, BC, CD, dan DA.

Menggunakan Teorema Pythagoras kita punya 52  a2  c2 , 160  b2  c2 , dan 9  a2  d 2 .


Dengan demikian CP 2  b 2  d 2  b 2  c 2  a 2  d 2   a 2  c 2   160  9  25  144 , sehingga

panjang CP  144  12 . .

Contoh
Diketahui gambar berikut. ABCD adalah persegi, dan E adalah titik
sembarang di luar persegi ABCD. Selidiki apakah berlaku hubungan
AE 2  CE 2  BE 2  DE 2 pada gambar di samping! (OSN)
Jawaban:
Kita tarik garis dari E tegak lurus AB dan DC seerta memotong AB dan DC berturut-turu di P dan
Q.

Jelas bahwa AQ  DP dan DB  PC . Misal sisi persegi ABCD adalah x, panjang AQ  a ,


QB  b , dan PE  c . Dengan menggunakan Teorema Pythagoras, kita punya

AE 2  AQ 2  QE 2  a 2   x  c  , BE 2  BQ 2  QE 2  b 2   x  c  , CE 2  CP2  PE 2  b2  c 2
2 2

, dan DE 2  DP2  PE 2  a 2  c 2 . Dengan demikian kita punya

AE 2  CE 2  a 2   x  c   b 2  c 2  b 2   x  c   a 2  c 2  BE 2  DE 2 .
2 2
.

B. Lingkaran

Tentunya kita telah mengenal lingkaran di materi sekolah biasa. Di sini kita akan menggunakan
pengetahuan lingkaran yang didapat disekolah untuk mengetahui hubungan lingkaran dengan
segitiga yaitu lingkaran dalam dan lingkaran luar suatu segitiga. Keliling dan luas suatu
lingkaran yang berjari-jari r berturut-turut adalah K  2 r dan L   r 2 . Jangan lupakan juga
materi tentang sudut keliling dan sudut pusat.

Contoh
Di dalam suatu lingkaran yang berjari-jari 4 cm dibuat persegi ABCD, sehingga titik sudut
persegi tersebut berada pada lingkaran. Luas persegi ABCD adalah .....
a. 64 cm2 c. 16 cm2 e. 4 cm2
b. 32 cm2 d. 8 cm2
(OSK 2003)
Jawaban:
Karena keempat titik sudut persegi terletak pada lingkaran maka panjang diagonal persegi sama
dengan diameter lingkaran yaitu 2 4  8 cm, sehingga luas persegi adalah
1 1
L  .d .d  .8.8  32 cm2. (ingat bahwa setiap persegi merupakan belah ketupat sehingga kita
2 2
dapat menggunakan rumus luas belah ketupat). (B). .

Contoh
Diketahui bentuk gambar di bawah berikut ini.

Titik-titik pusat lingkaran B, C, D, dan E diletakkan pada garis tengah lingkaran A dan garis
tengah lingkaran B sama dengan jari-jari lingkaran A. Lingkaran C, D, dan E sama besar dan
sepasang-sepasang bersinggungan di luar sehingga jumlah panjang garis tengah ketiga lingkaran
tersebut sama dengan jari-jari lingkaran A. Bagaimanakah perbandingan keliling lingkaran A
dengan jumlah keliling lingkaran B, C, D, dan E? (OSN 2003)
Jawaban:
Misalkan panjang-jar-jari lingkaran A adalah r satuan, maka panjang jari-jari lingkaran B adalah
1 1 1
r satuan dan panjang jari-jari lingkaran C, D, dan E adalah r / 3  r satuan. Keliling
2 2 6
linkaran A  K A  2 r dan jumlah keliling lingkaran B, C, D, dan E adalah

1 1 1 1 
K B  KC  K D  K E  2  r  r  r  r    r . Jadi keliling lingkaran A sama dengan
2 6 6 6 
jumlah keliling lingkaran B, C, D, dan E. .
Contoh
Pada gambar dibawah, titik O adalah pusat lingkaran yang berjari-jari r. Jika panjang ruas garis
1
ED juga sama dengan r, buktikanlah bahwa DEC  AOB .
3

(OSP 2005)
Jawaban:
Misal AOB  x0 dan DEC  y 0 . Karena DOE sama kaki dengan DE  DO maka

DOC  y 0 , akibatnya ODB  2 y 0 . Di samping itu ODB sama kaki sehingga

OBD  ODB  2 y 0 dan DOB  1800  4 y 0 . Perhatikan bahwa kita punya


1 1
COD  DOB  BOA  1800 , artinya y  180  4 y  x  180  y  x  DEC  AOB .
3 3
Terbukti. .

Lingkaran dalam suatu segitiga adalah lingkaran yang berada di dalam segitga dan
menyinggung ketiga sisi segitiga tersebut. Titik pusat lingkaran dalam ini merupakan
perpotongan ketiga garis bagi sudut-sudut segitiga tersebut.

Panjang jari-jari lingkaran dalam segitiga adalah r 


 ABC  .
s
Lingkaran luar suatu segitiga adalah lingkaran yang melalui ketiga titik sudut segitiga tersebut.
Titik pusat lingkaran luar ini merupakan perpotongan ketiga garis sumbu sisi segitiga.

abc
Panjang jari-jari lingkaran luar segitiga adalah R  , dengan a, b, dan c menyatakan
4  ABC 

panjang sisi-sisi dari segitiga ABC.

Contoh
Lingkaran M adalah lingkaran dalam dari ABC , sedangkan lingkaran N merupakan
lingkaran dalam dari ACD . Lingkaran M dan N bersinggungan di titik E . Jika panjang sisi
AD  x cm, AB  y cm, BC  z cm , tentukan panjang sisi DC (nyatakan dalam x, y dan z .)

(OSN 2008)
Jawaban:
Misalkan lingkaran M menyinggung sisi AB dan BC berturut-turut di P dan Q, sedang lingkaran
N menyinggung sisi AD dan DC berturut-turut di R dan S.

Jelas bahwa DR  DS (keduanya garis singgung lingkaran N) dan BP  BQ (keduannya juga


merupakan garis singgung lingkaran M).
Perhatikan AR dan AE merupakan garis singgung lingkaran N yang ditarik dari A sehingga
AR  AE , tetapi AE dan AP merupakan garis singgung lingkaran M yang ditari dari A sehingga
AE  AP . Dari sini kita peroleh AR  AP . Dengan cara yang sama kita juga punya CS  CQ .
Dengan demikian, kita punya AR  RD  BQ  QC  AP  DS  BP  SC  AP  PB  DC  SC
yang selanjutnya kita peroleh AD  BC  AB  CD , sehingga DC  AD  BC  AB  x  z  y .
Jadi panjang DC  x  z  y . .

C. Bangun Ruang

Bangun ruang yang biasanya muncul dalam olimpiade diantaranya kubus, balok, tabung, limas,
kerucut, dan bola. Terkadang soal menggabungkan beberapa bangun ruang tersebut dan kadang
kala kita diminta menghitung volume atau luas permukaan bangun terpancung. Untuk bangun
terpancung ini biasanya kita menggunakan kesebangunan. Di sini kita tidak akan membahas
rumus-rumus volume maupun luas permukaan karena sudah kita pelajari di materi sekolah. Kita
akan mencoba melihat teknik-teknik pengerjaan soal olimpiade baik OSK, OSP, maupun OSN.
Contoh
Perhatikan Gambar 4(a) sebagai kubus sempurna dan Gambar 4(b) merupakan kubus yang sama
dengan Gambar 4(a) dengan salah satu titik sudut dipotong dengan potongan berbentuk limas.

Jika panjang rusuk kubus 6a cm dan panjang rusuk tegak limas 2 1 a cm, maka volum bangun
3
baru adalah ….

(OSP 2009)
Jawaban:

Volume kubus   6a   216a 3 , sekarang akan kita hitung volume limas yang dibuang. Pertama
3

akan dihitung tinggi limas. Tinggi limas diproyeksikan ke alas tepat di titik berat segitiga
(alasnya) di mana perbandingannya antara yang pendek dengan yang panjang adalah 1:2.
7 7
Panjang garis berat segitiga dengan sisi a adalah a 3 , sehingga tinggi limas
3 6
2 2 2 2 2
7  2 7  7  7  7  7 14
t   a  a   a  a   a 9 1  a 8 2. Dengan
3  36  3  9  9  9 9
2
1 1 7  14 686 3
demikian, volume limas Vlimas  Lalas  t  .  a  3. a 2 a 6 . Jadi volume
3 3 3  9 243

 686 3 
bangun baru adalah V  Vkubus  Vlimas   216a 3  a 6  cm3. .
 243 
Contoh
Sebuah ember terbuat dari seng seperti tampak pada gambar. Luas seng yang digunakan untuk
ember tersebut adalah ……

(OSP2006)
Jawaban:
Luas alasnya  Lalas   .152  225 cm2. Sekarang akan kita hitung luas selimut ember. Ember
tersebut merupakan kerucut terpancung, sehingga kalau kita buat kerucut lengkapnya dalah
sebagai berikut.

Misalkan panjang garis tinggi dan garis pelukis kerucut kecil berturut-turut adalah t dan s , maka
t 15
dengan menggunakan kesebangunan kita punya   5t  3t  120  t  60 , sehingga
t  40 25

panjang s  602  152  15 17 dan panjang garis pelukis kerucut besar  602  252  65 .
Dengan demikian, luas selimut ember sama dengan luas selimut kerucut besar dikurangi dengan

 
luas selimut kerucut kecil yaitu Lselimut   .25.65   .15.15 17  25 65  9 17 . Jadi luas seng

yang digunakan untuk ember tersebut adalah

    
L  Lalas  Lselimut  225  25 65  9 17  25 9  65  9 17  25 74  9 17 cm2.  .
h

Teori Bilangan

Teori bilangan merupakan salah satu materi dalam olimpiade. Pada bab ini, kita akan
mempelajari tentang konsep dasar keterbagian, algoritma pembagian, faktor
persekutuan terbesar, dan kelipatan persekutuan terkecil, serta persamaan Diophantine.
Namun sebelumnya, akan dijelaskan terlebih dahulu bagaimana cara menyajikan
bilangan asli dengan menggunakan basis.

A. Penyajian Bilangan Bulat

Pada saat kita masih di sekolah dasar, tentu saja kita pernah menjumpai soal yang
dilontarkan oleh guru kita sebagai berikut:
1. Berapakah angka ratusan dari 3356?
2. Angka 4 pada bilangan 4.777 bernilai?
Nah, pertanyaan di atas dimaksudkan pada basis 10, yakni ada satuan, puluhan, ratusan,
ribuah, puluh ribuan, dst. Sebagai contoh bilangan 3.356 dimaksudkan 3 ribuan, 3
ratusan, 5 puluhan, dan 6 satuan atau secara metematika ddapat kita nyatakan sebagai
3356  3 1000  3 100  5 10  6  3.103  3.102  5.10  6 .
Penyajian di atas, disebut penyajian dalam basis 10 atau lebih dikenal dengan disebut
sistem desimal. Angka-angka/digit-digit yang digunaan dalam basis 10 adalah 0, 1, 2,
3, 4,…, 9. Perlu diingat bahwa angka hanyalah symbol, sedangkan bilangan adalah nilai
dari angka-angka yang kita bentuk. Secara umum, untuk sebarang bilangan bulat b  1
kita dapat menyajikan bilangan dalam basis b. Angka-angka yang digunakan dalam
basis b ini adalah anggota-anggota dari himpunan{0,1,2…b-1}. Penulisan basis,
biasanya dituliskan di sebelah kanan bawah bilangan. Sebagai conntoh 200911 adalah
2009 dalam basis 11. Jika bilangan basisnya tidak dituliskan maka bilangan yang
dimaksud tersebut adalah bilangan dalam basis 10. Sebagai contoh jika kita menemukan
bilangan 2009 maka yang dimaksud adalah 200910.

1
h

Secara matematika, nilai dari bilangan yang disajikan dalam basis b adalah sebagai
berikut: nilai dari anan-1…a1a0b dengan ai  {0,1,2…b-1} untuk setiap i=1,2,3…n dalam
basis 10 adalah
an an 1...a1a0 b  anb n  an 1b n 1  ...  a1b  a0

Pernyataan ini sekaligus menjelaskan kepada kita bagaimana mengubah bilangan


menjadi basis 10.

Contoh
Jika P, Q, R adalah angka-angka dari suatu bilangan dan
(100P  10Q  R)( P  Q  R)  2008 ,
Maka nilai Q adalah.....(OSK 2008)
a. 3 d. 6
b. 4 e. 7
c. 5
Jawaban:
Perhatikan bahwa 100 P  10Q  R  PQR dan 2008  251 8 atau 2008  502  4 .

 
Jika PQR  251 maka PQR  P  Q  R   251 8  2008 (memenuhi), sedangkan jika

 
PQR  502 maka PQR  P  Q  R   502  7  2008 (tidak memenuhi). Jadi nilai Q

adalah 5. (C).

Contoh
Diketahui N  9  99  999  ...  999...9
121 angka

Tentukan nilai N. (OSN 2006)


Jawaban:
Untuk setiap bilangan asli n kita punya 999...9  1000...0  1 , sehingga
n angka n angka 0

2
h

N  9  99  999  ...  999...9


121 angka

 
 10  1  100  1  1000  1  ...  1000...0  1 
 
 121 angka 0 
 
 10  100  1000  ...  1000...0   121
 
 121 angka 0 

 111...1 0  121
121 angka 1

 111...1 0000  1110  121


119 angka 1

 111...1 0000  989


119 angka 1

 111...1 0989.
119 angka 1

B. Keterbagian

Manakah di antara bilangan-bilangan 1, 2, 3, 4, 5, …, 100 yang habis dibagi 2? Dengan


mudah tentu kita dapat menjawab bahwa yang habis dibagi 2 adalah bilangan-bilangan
genap. Apa itu habis membagi? Apa itu habis dibagi? Ini akan dijelaskan pada sub bab
ini.

Definisi B.1
Bilangan bulat a habis membagi b ditulis a|b jika dan hanya jika terdapat bilangan bulat
k sehingga b=ka. Dalam hal a tidak habis membagi b dituliskan dengan .
Keterangan:
Selain a|b dibaca a habis membagi b, juga dapat dibaca a faktor dari b, atau b habis
dibagi a, atau b kelipatan a.

Contoh
1. Kita dapat mengatakan 7|98 karena terdapat bilangan bulat yakni 14 sehingga
98=14 × 7.
2. Kita dapat mangatakan karena kita akan pernah dapat mencari bilangan
bulat k sehingga 3  2  k .

3
h

Contoh
8
Tentukan semua bilangan asli n sehingga adalah bilangan bulat!
n
Jawaban:
8
agar bulat maka n|8,sehingga n merupakan faktor dari 8, sehingga n yang mungkin
n
adalah n=1,2,4,atau 8.

Dari definisi di atas kita dapat menurunkan sefat-sifat sebagai berikut


Sifat B.1
Untuk setiap bilangan bulat a yang tidak nol selalu berlaku a|a dan a|0
Sifat B.2
Untuk setiap bilangan bulat a selalu berlaku 1|a
Sifat B.3
Jika a|b maka ac|bc untuk setiap bilangan bulat c yang tidak nol.
Sifat B.4
Jika a|b maka a  b .

Sifat B.5
Jika a|b dan a|c maka a|(mb+nc) untuk setiap bilangan bulat m dan n.

Di sini, kita hanya akan membuktikan Sifat B.5. Sedangkan untuk sifat-sifat yang
lainnya diserahkan kepada pembaca sebagai latihan.

Bukti Sifat B.5


Perhatikan bahwa a|b artinya terdapat bilangan bulat k sehingga b=ka, dan juga kita
tahu a|c yang berarti terdapat bilangan bulat l sehingga c=la. Dari kedua fakta tersebut
kita punya mb+nc=mka+nla=(mk+nl)a yang berarti a|(mb+nc) ■

Perhatikan bahwa pada sifat B.5, jika m dan n berturut-turut kita ganti dengan 1 dan -1
serta 1 dan 1 maka akan kita perleh Jika a|b dan a|c maka a|b-c dan a|b+c. Kita akan

4
h

sering menggunakan sifat-sifat di atas untuk menyelesaikan masalah. Sekarang marilah


kita lihat beberapa contoh soalnya.

Contoh
Diketahui a, b, c, dan d adalah bilangan asli. Jika c habis dibagi a, dan d habis dibagi b,
maka pernyataan berikut:
(i) cd habis dibagi ab
(ii) c +d habis dibagi a+b
(iii) cd habis dibagi a
(iv) bc habis dibagi ab
(v) dc habis dibagi ba
yang selalu benar adalah ....
a. hanya (i) d. semuanya, kecuali (v)
b. hanya (i), (iii), dan (iv) e. semuanya
c. semuanya, kecuali (ii)
(OSP 2007)
Jawaban:
Dikatahui bahwa a|c dan b|d. Jelas bahwa ab|cd, a|cd, dan ab|bc, jadi (i), (iii), dan (iv)
selalu benar. Sekarang pilih a=1, b=1, c=1, dan d=2, maka jelas bahwa a|c dan b|d.
Akan tetapi, a+b=2 dan c+d=3 yang tentu saja , jadi (ii) tidak selalu benar.
Terakhir perhatikan bahwa a  c , sehingga b a | b c . Dengan demikian, karena b|d maka

b c | d c dan karena kita juga tahu bahwa b a | b c maka b a | d c yakni (v) selalu benar.
Jadi semua pernyataan selalu benar kecuali (ii). (C).

Contoh
n 1
H adalah himpunan semua bilangan asli n demikian sehingga bentuk
n3
menghasilkan bilangan bulat kurang dari 1, maka banyaknya himpunan bagian tak
kosong dari H adalah ... (OSK 2005)
Jawaban:

5
h

n 1
Agar merupakan bilangan bulat haruslah n  3| n 1 , di lain pihak kita juga punya
n3
n  3| n  3 . Dengan demikian n  3 | (n  1)   n  3  atau ekivalen dengan n  3 | 2 . Dari

sini kita simpulkan n  3  1,1, 2 , atau 2, yang selanjutnya kita dapatkan solusi
n  2, 4,1, atau 5. Sekarang kita periksa mana yang hasilnya kurang dari 1.
n 1 2 1
 n=2, maka   1  1 (memenuhi),
n3 23
n 1 4 1
 n=4, maka   3  1 (tidak memenuhi),
n3 43
n 1 1 1
 n=1, maka   0  1 (memenuhi),
n  3 1 3
n 1 5 1
 n=5, maka   2  1 (tidak memenuhi).
n3 53
Jadi banyak anggota H adalah 2, sehingga banyaknya himpunan bagian tak kosong dari
H adalah 22  1  3 . (dikurangi dengan 1 karena himpunan tak kosong tidak masuk). .

TIPS:
Untuk menentukan n sehingga f (n) | ( g (n) manipulasilah sampai menemukan
bentuk f (n) | m dengan m adalah suatu bilangan bulat. Sehingga dapat kita
simpulkan f(n) adalah faktor-faktor dari m.

Dari sifat-sifat keterbagian di atas, kita dapat mentukan kriteria suatu bilangan yang
habis dibagi 2, 3, 4, 5, 8, 9, dan 11.

Teorema B.1 (kriteria habis dibagi)

Diketahui m  an an 1...a2 a1 adalah bilangan asli n digit.


1. m habis dibagi 2 jika dan hanya jika digit terakhirnya habis dibagi 2 atau dengan
kata lain jika digit satuannya genap.
2. m habis dibagi 4 jika dan hanya jika dua digit terakhirnya habis dibagi 4.
3. m habis dibagi 8 jika dan hanya jika tiga digit terakhirnya habis dibagi 8.

6
h

4. m habis dibagi 5 jika dan hanya jika digit terakhirnya habis dibagi 5 atau dengan
kata lain jika digit satuannya 0 atau 5.
5. m habis dibagi 3 jika dan hanya jumlah semua digit-digitnya habis dibagi 3.
6. m habis dibagi 9 jika dan hanya jumlah semua digit-digitnya habis dibagi 9.
7. m habis dibagi 11 jika dan hanya jumlah selang-seling digit-digitnya habis dibagi
11 yakni a1  a2  a3  ...an habis dibagi 11.
Bukti:

1. 2 | an an 1...a2 a1  2 | an an 1...a2 0  a1  2 | a1 .

2. 4 | an an 1...a2 a1  4 | an an 1...a3 00  a2a1  4 | a2a1.

3. 8 | an an 1...a2 a1  8 | an an 1...a4 000  a3a2a1  8 | a3a2a1.

4. 5 | an an 1...a2 a1  5 | an an 1...a2 0  a1  5 | a1  a1  0 atau 5.

5. an an 1...a2 a1   an  an 1  ...  a2  a1   99...9 an  99...9 an  2  ...  9a2 sehingga


n 1 kali n  2 kali

3 | an an 1...a2 a1   an  an 1  ...  a2  a1  . Dari sini dapat kita simpulkan bahwa

3 | m  3 |  an  an 1  ...  a2  a1 

6. an an 1...a2 a1   an  an 1  ...  a2  a1   99...9 an  99...9 an  2  ...  9a2 sehingga


n 1 kali n  2 kali

9 | an an 1...a2 a1   an  an 1  ...  a2  a1  . Dari sini dapat kita simpulkan bahwa

9 | m  9 |  an  an 1  ...  a2  a1 

7. an an 1...a2 a1   a1  a2  a3  ...an   11a2  99a3  1001a4  9999a5  ...an yang

akan ekivalen dengan


8. an an 1...a2 a1   a1  a2  a3  ...an   11a2  99a3  1001a4  9999a5  ...an sehingga

11| an an 1...a2 a1   a1  a2  a3  ...an  . Dari sini dapat kita simpulkan bahwa

11| m  11|  a1  a2  a3  ...an  .

C. Algoritma Pembagian

Algoritma pembagian juga merupakan modal dasar yang sangat penting dalam teori
bilangan. Pertama akan dipelajari teorema dasar algoritma pembagian:

7
h

Teorema C.1
Jika a dan b adalah bilangan bulat dan b>0, maka terdapat dengan tunggal bilangan
bulat q dan r sehingga
a  bq  r
dengan 0  r  b .
Bukti:
Pandang himpunan ..., a -3b, a - 2b, a - b, a, a  b, a  2b, a  3b ,... Jika barisan tersebut
memuat unsur nol, maka terdapat bilangan bulat q sehingga a  bq  r dengan r=0. Jika
barisan tersebut tidak terdapat unsur nol, maka a tidak mungkin nol. Jika a>0 maka
a  ab  a(1  b)  0 , dan jika a<0 maka a - ab  -a(b -1)  0 . Jadi, barisan tersebut
memuat unsur positif. Dengan demikian, jika kita himpun semua elemen yang positif
sebut saja himpunan S, maka S mempunyai elemen terkecil, sebut elemen minmal
tersebut adalah r  a - qb . Kita akan buktikan bahwa r<b. Jelas r≠b (mengapa?),
andaikan r>b maka akan kita peroleh s  a - (q  1)b  a - qb - b  r - b  0 . Perhatikan
bahwa s∈S, dan s<r. Ini kontradiksi dengan asumsi bahwa r merupakan elemen terkecil.
Jadi haruslah terdapat bilangan bulat q sehingga
0< r  a - qb <b
atau dengan kata lain
a  bq  r dengan 0<r<b .

Sekarang akan kita buktikan ketunggalannya. Misalkan terdapat bilangan bulat


0≤r₁,r₂<b dan q₁ serta q₂ sehingga a=bq₁+r₁=bq₂+r₂. Dari sini akan diperoleh
b(q₁-q₂)=r₂-r₁ yang berarti b|(r₂-r₁), akan tetapi -b<r₂-r₁<b , akibatnya r₂-r₁=0 atau
dengan kata lain r₂=r₁. Dengan fakta r₂=r₂ ini juga akan berakibat q₁=q₂ dan kita
selesai.

8
h

Selanjutnya untuk q disebut bagian bulat hasil bagi a oleh b dan r disebut dengan sisa
pembagian a oleh b. Perlu diperhatikan bahwa 0  r  b . Khusus jika r  0 maka kita
akan punya b | a .

Dari teorema di atas, dapat kita pahami bahwa jika m suatu bilangan asli, maka untuk
sebarang bilangan bulat n dapat dinyatakan sebagai
n=mk+r
untuk suatu bilangan bulat k dan r dengan 0≤r≤m-1. Bilangan yang berbentuk mk+r
adalah bilangan bulat yang bersisa r ketika dibagi m. Sebagai contoh, jika kita ambil
m=2, maka fakta di atas mengatakan bahwa setiap bilangan bulat dapat dinyatakan
dalam bentuk 2k atau 2k+1, yang selanjutnya dalam kehidupan kita sehari-hari bilangan
yang berbentuk 2k dan 2k+1 berturut-turut kita katakan bilangan genap dan bilangan
ganjil. Jika kita ambil m=3, maka fakta di atas mengatakan bahwa setiap bilangan bulat
habis dibagi 3, bersisa 1, atau bersisa 2 ketika dibagi 3.

Sekarang, mari kita lihat beberapa contoh berikut.

Contoh
311  1
Jika dibagi 9, maka sisanya adalah… (OSK 2008)
2
a. 2 d. 6
b. 3 e. 8
c. 4
Jawaban:

311  1  3  1  3  3  ...  3  3  1 10 9
10 9 2

Perhatikan bahwa   3  3  ...  32  3  1 . Kita


2 2
tulis 310  39  ...  32  3  1  9  38  37  ...  1  3  1 , sehingga sisanya jika dibagi 9

adalah 4. (C).

Contoh

9
h

Diberikan dua bilangan bulat yang berjumlah 37. Jika bilangan yang lebih besar
dibagi dengan bilangan yang lebih kecil, maka hasil baginya adalah 3 dan sisanya 5.
Selisih kedua bilangan tersebut adalah .... (OSP 2007)
a. 3 d. 21
b. 5 e. 29
c. 8
Jawaban:
Misalkan bilangan tersebut adalah a dan b dengan a  b . Dari yang diketahui kita
punya a  b  37 dan a  3b  5 . Dari sini kita peroleh 3b  5  b  37  b  8 ,
sehingga selisih kedua bilangan tersebut adalah a  b  3b  5  b  2b  5  21 . (D).

D. Faktor Persekutuan terbesar dan Kelipatan Persekutuan terkecil

Pada saat sekolah dasar, kita semua tentu telah mengenal pembagi sekutu terbesar atau
biasa disebut faktor persekutuan terbesar (FPB), atau disebut juga greatest common
divisor(gcd).

Definisi D.1.
Diberikan a dan b adalah bilangan bulat yang tidak keduanya nol. Bilangan asli d
disebut faktor persekekutuan terbesar dari a dan b atau ditulis dengan d =gcd(a,b) jika
(i). d|a dan d|b,
(ii). untuk setiap bilangan asli c dengan c|a dan c|b haruslah berlaku c  d .

Pada definisi di atas, bagian (i) mengatakan bahwa d adalah pembagi sekutu dari a dan
b, sedangkan bagian (ii) mengatakan bahwa untuk setiap pembagi sekutu dari a dan b
harus lebih kecil atau sama dengan d,dengan kata lain (ii) mengatakan bahwa d
merupakan pembagi sekutu yang terbesar.

Definisi D.2.
Bilangan bulat a dan b dikatakan saling prima (relatif prima) jika gcd(a,b)=1.

Definisi D.3.

10
h

Untuk sebarang bilangan bulat a,b dan c didefinisikan


gcd(a, b, c)  gcd(gcd(a, b), c)  gcd(a,gcd(b, c)) .

Dari definisi di atas, dapat diturunkan beberapa sifat di bawah ini:


1. gcd(a, b)  gcd(b, a)  gcd(| a |,| b |) ,
2. gcd(a,1)  1 untuk setiap bilangan bulat a,
3. gcd(a,0) | a | untuk setiap bilangan bulat taknol a,
4. gcd(ma, mb)  |m | gcd(a, b) untuk setiap bilangan bulat tak nol m,a dan b.

a b
5. jika d  gcd(a, b) maka gcd  ,   1 .
d d 
Di sini hanya akan kita buktikan untuk sifat (5) saja, sedangkan untuk bukti sifat-sifat
yang lain diserahkan kepada pembaca sebagai latihan.

Bukti sifat 5
a b
Karena d  gcd(a, b) maka jelas dan merupakan bilangan bulat. Misalkan
d d
a b a b
g  gcd  ,  , maka g | dan g | yang selanjutnya kita peroleh gd | a dan gd | b.
d d  d d
Dari sini kita punya bahwa gd adalah sebarang faktor persekutuan dari a dan b. Oleh
karena d  gcd(a, b) maka gd  d  g  1 , sehingga haruslah g  1 .

Contoh
Misalkan n adalah bilangan asli yang tidak lebih dari 24, maka jumlah dari semua nilai
yang memenuhi agar n dan 24 relatif prima adalah ....
a. 120 c. 95 e. 81
b. 96 d. 82
(OSK 2008)
Jawaban:
Yang relatif prima dengan 24 adalah 1, 5, 7, 11, 13, 17, 19, dan 23. Sehingga jumlah
semuanya adalah 96. (B).

Menentukan gcd dua bilangan dengan Algoritma Euclide

11
h

Ada beberapa cara untuk menghitung FPB dari dua bilangan. Cara yang tentu kita
semua tahu adalah dengan cara mendaftar faktor-faktor masing-masing bilangan
kemudian dicari faktor yang sama, dan yang kedua adalah dengan pohon faktor. Kedua
cara ini akan sangat sulit apabila kita menghitung bilangan yang faktor primanya cukup
besar. Berikut ini akan dijelaskan cara alternatif menghitung FPB yaitu dengan
menggunakan Algoritma Euclid.

a dan b adalah bilangan bulat yang tidak keduanya nol. Kita akan menghitung gcd dari a
dan b dengan menggunakan algoritma pembagian yang telah kita pelajari sebelumnya.
Karena gcd(a, b)  gcd(b, a)  gcd(| a |,| b |) , maka di sini hanya akan dibahas untuk a
dan b bilanganasli dengan a  b .
Berdasarkan algoritma pembagian, akan terdapat bilangan bulat q dan r dengan
0  r  b sehingga a  bq+r atau ekivalen dengan r  a  bq . Perhatikan bahwa untuk
setiap pembagi sekutu a dan b pasti merupakan pembagi dari r. Oleh sebab itu, dapat
kita simpulkan gcd(a, b)  gcd(b, r ).
Jika r  0, maka gcd(a, b)  gcd(b,0)  b. Jika r  0 kita dapat lakukan langkah yang

sama pada b dan r, yakni terdapat q dan r dengan 0  r1  r sehingga b  rq1  r1 .


Dengan argumentasi yang sama seperti sebelumnya, kita simpulkan
gcd(b, r )  gcd(r, r1 ). Jika r =0, maka gcd(b, r )  gcd(r, r1 )  r . Jika tidak, kita dapat

melakukan langkah di atas sehingga kita peroleh barisan tak hingga r1 , r2 , r3 ,... Akan

tetapi, karena a dan b berhingga, maka tentu akan terdapat n sehingga r0  0 . Dengan
demikian
gcd(a, b)  gcd(b, r )  gcd(r, r1 )  gcd(r1; r2 )  ...  gcd(rn-1, rn )  gcd(rn-1;0)  rn-1.

Untuk lebih jelasnya, perhatikan contoh berikut.

Contoh
Hitung gcd(2008,123456).
Jawab:

12
h

gcd(2008,123456)  gcd(123456, 2008). Dengan algoritma pembagian


123456  61 2008  968 , sehingga gcd(123456, 2008)  gcd(2008,968). Kemudian
2008  2  968  72 , sehingga gcd(2008,968)  gcd(968,72), seterusnya dengan
algoritma pembagian akan kita peroleh
gcd(968,72)  gcd(72,32)  gcd(32,8)  gcd(8,0)  8 .

Identitas Bezout
Jika d  gcd(a, b) maka terdapat bilangan bulat x dan y sehingga ax  by  d .
Bukti:
Bentuk himpunan S  ax  by | x, y  , ax  by  0 .

Perhatikan bahwa jika kita ambil x  a dan y  b maka kita punya a  b  0 (ingat
pendefinisian gcd mengasumsikan a dan b tidak keduanya nol) yang berarti S tidak
kosong. Dengan demikian S mempunyai elemen terkecil, sebut saja d. Kita akan
buktikan bahwa d  gcd(a, b). Pertama akan kita buktikan bahwa d|a dan d|b. Dengan
algoritma pembagian kita dapat tulis: a  dq  r dengan 0  r  d atau dengan kata lain
r  a  dq . Akan tetapi d  S yang berarti d  am  bn untuk suatu m, n  . Oleh
karena itu, kita punya
r  a  dq  a  (am  bn)q  a  aqm  bnq  a(1  qm)  b(nq)  S .
Karena d adalah elemen terkecil dari S dan 0  r  d maka r  0 yang berarti d|a.
Dengan cara yang sama kita peroleh juga d|b. Sekarang misalkan c adalah sebarang
bilangan asli dengan c|a dan c|b, maka c | am  bn atau c | d .
Terbukti bahwa d  gcd(a, b).

Perhatikan bahwa dari pembuktian identitas Bezout, kita tahu bahwa jika d  gcd(a, b)
maka untuk sebarang faktor persekutuan dari a dan b pastilah faktor persekutuan
tersebut habis membagi d. Di samping itu, kita juga akan punya dua sifat berikut ini.

Teorema D.1.
Jika a|bc dan gcd(a, b)  1 maka a | c.
Bukti:

13
h

Menurut Identitas Bezout, terdapat bilangan bulat x dan y sehingga ax  by  1 ,


sehingga kita punya acx  bcy  c  acx  c  bcy . Akibatnya a | c  bcy , tetapi a | bcy ,
sebagai konsekuensinya haruslah a | c.

Teorema D.2.
Jika a | c, b | c, dan gcd  a, b   1 maka ab | c .

Bukti:
b | c artinya ada bilangan bulat k sehingga c  kb . Perhatikan bahwa a | c  a | kb ,
sehingga karena gcd(a, b)  1 maka menurut teorema D.1 haruslah a | k . Dari sini kita
peroleh ab | kb  ab | c .
Contoh
Dua bilangan bulat m dan n dikatakan relatif prima jika ada bilangan bulat a dan b
sedemikian sehingga am  bn  1 . Tunjukkan bahwa untuk setiap bilangan bulat p,
pasangan bilangan yang dibentuk oleh 21 p  4 dan 14 p  3 senantiasa relatif prima.
(OSN 2006)
Jawaban:
Pilih a  2, b  3 maka kita punya 2  21 p  4   3 14 p  3   1 yang menurut definisi

berarti 21 p  4 dan 14 p  3 relatif prima.

Selain pembagi sekutu terbesar, pada saat berada di sekolah dasar juga kita telah
mengenal kelipatan persekutuan terkecil (KPK). Dalam pembahasan selanjutnya, untuk
sebarang biangan bulat a dan b KPKdari a dan b kita tulis dengan [a,b].

Definisi D.4
Diberikan a dan b adalah bilangan bulat yang tidak keduanya nol. Bilangan bulat tak
negatif m disebut kelipatan persekekutuan terkecil dari a dan b atau ditulis dengan
m   a, b  jika

(i). m|a dan m|b,


(ii). untuk setiap bilangan asli n dengan a|n dan b|n haruslah berlaku m  n .

14
h

Pada definisi di atas, bagian (i) mengatakan bahwa m adalah kelipatan sekutu dari a dan
b, sedangkan bagian (ii) mengatakan bahwa untuk setiap kelipatan sekutu dari a dan b
harus lebih besar atau sama dengan m, dengan kata lain (ii) mengatakan bahwa m
merupakan kelipatan persekutuan yang terkecil.

Definisi D.5.
Untuk sebarang bilangan bulat a,b dan c didefinisikan

 a, b, c    a, b  , c   a, b, c  .

Dari definisi di atas, dapat diturunkan beberapa sifat di bawah ini.


1.  a, b    b, a   | a |,| b |  , untuk setiap bilangan bulat a dan b yang tidak keduanya 0.

2.  a,1 | a | untuk setiap bilangan bulat a,

3.  a, 0   0 untuk setiap bilangan bulat taknol a,

4.  ma, mb   |m |  a, b  untuk setiap bilangan bulat tak nol m.

Tentunya kita dapat menghitung KPK dari dua bilangan bulat dengan mendaftar
masing-masing kelipatannya ataupun dengan pohon faktor. Akan tetapi, jika kita sudah
mengetahui FPB dari kedua bilangan tersebut maka dengan mudah kita dapat mencari
KPKnya dengan menggunakan hubungan yang dijelaskan pada teorema berikut ini.

Teorema D.3
Jika a dan b adalah bilangan bulat yang tidak keduanya nol, maka
ab
 a, b   .
gcd  a, b 

Bukti:
Misalkan d  gcd(a, b) , maka a dan b dapat kita nyatakan sebagai a  dx dan b  dy
untuk suatu bilangan bulat x dan y dengan gcd( x, y)  1. Selanjutnya didefinisikan
m  dxy . Akan kita buktikan bahwa m  [a, b]. Jelas bahwa a | m dan b | m . Sekarang,

15
h

ambil sebarang bilangan asli n dengan sifat a | n dan b | n , artinya n  ka dan n  lb


untuk suatu bilangan bulat k dan l. Dari sini kita dapatkan
ka  lb  kdx  ldy  kx  ly .
Kitapunya x | ly , dan karena gcd( x, y)  1 maka x | l yang berarti l  tx untuk suatu
bilangan bulat t. Dengan demikian kita n  lb  ldy  tdxy  tm akibatnya m | n dan kita
selesai membuktikan m  [a, b].

Contoh
Ada berapa banyakkah bilangan asli yang tidak lebih besar dari 2004 yang bersisa 1
ketika dibagi 2, bersisa 2 ketika dibagi 3, bersisa 3 ketika dibagi 4, dan bersisa 4 ketika
dibagi 5? (OSN 2004)
Jawaban:
Misalkan n adalah bilangan yang memenuhi semua syarat di atas. Bilangan n bersisa 1
ketika dibagi 2 artinya n  2a 1 , bersisa 2 ketika dibagi 3 artinya n  3b 1 , bersisa 3
ketika dibagi 4 artinya n  4c 1 , dan bersisa 4 ketika dibagi 5 artinya n  5d 1 . Agar
n berbentuk n  2a 1 , n  3b 1 , n  4c 1 , dan n  5d 1 secara bersama-sama
maka n  tk 1 dengan t   2,3, 4,5  60 . Dengan demikian bilangan yang memenuhi

syarat di atas adalah bilangan yang berbentuk 60k  1 untuk sebarang bilangan bulat k.
Karena n merupakan bilangan asli maka jelas bahwa k  1 , di samping itu ada syarat
25
n  2004  60k  1  2004  60k  2005  k  33 , tetapi karena k bulat maka
60
k  33 . Jadi 1  k  33 , yang berarti ada sebanyak 33 bilangan asli yang memenuhi.

E. Bilangan Prima

Bilangan prima dapat dikatakan bilangan yang paling sederhana. Kita dapat dengan
mudah mencari informasi dari bilangan ini seperti banyak faktor, jumlah semua faktor,
FPB dan KPK dengan bilangan lain, dan yang lainnya.

Definisi E.1.

16
h

Bilangan bulat positif p dikatakan bilangan prima jika p mempunyai tepat dua faktor
positif yaitu 1 dan p sendiri.

Definisi E.2.
Bilangan bulat positif n dikatakan bilangan komposit jika n mempunyai lebih dari 2
faktor positif.

Definisi 2 di atas juga dapat kita katakana bahwa n adalah bilangan komposit jika
terdapat bilangan bulat positif a, b  1 sehingga n  ab.

Contoh
Himpunan semua bilangan prima yang kurang dari seratus dan kuadrat bilangan tersebut
ditambah dua juga merupakan bilangan prima adalah …. (OSK 2007)
Jawaban:
Jika p  3 maka p2  2  13 juga prima.
Jika p  3 maka menurut algoritma pembagian, p berbentuk 3k  1 atau 3k  2 untuk

suatu bilangan bulat k. Untuk n  3k  1 kita punya n 2  2   3k  1  2  9k 2  6k  3


2

habis dibagi 3, dan untuk n  3k  2 kita punya n 2  2   3k  2   2  9k 2  12k  6


2

yang juga habis dibagi 3. Dengan demikian satu-satunya bilangan prima yang mungkin
hanyalah 3. Jadi himpunan yang dimaksud adalah 3 .

Teorema E.1.
Banyak bilangan prima adalah tak hingga.
Bukti:
Andaikan hanya ada sejumlah berhingga bilangan prima, sebut saja p1 , p2 ,..., pn dengan

p1  p2  ...  pn .

Bentuk N  p1 p2 ... pn  1 , jelas N  pn . Perhatikan bahwa untuk setiap k  1, 2,3,..., n

haruslah pk tidak membagi N karena jika pk | N maka pk |1 yang jelas tidak mungkin.

Dengan demikian, N prima atau terbagi oleh bilangan prima lebih dari pn . Hal ini
kontradiksi dengan pengandaian kita. Jadi ada tak hingga banyak bilangan prima.

17
h

Teorema E.3.
Jika p prima, maka untuk sebarang bilangan asli n berlaku p | n atau gcd  p, n   1 .

Bukti:
Misalkan d  gcd  p, n  , maka d | p dan d | n . Dari fakta d | p kita punya d  1 atau

d  p  p|n.

Teorema E.4.
Jika p prima dan p | ab untuk suatu bilangan bulat a dan b maka p | a atau p | b .
Bukti:
Jika p | a maka kita selesai, asumsikan p tidak habis membagi a, maka menurut

Teorema E.3 haruslah gcd  p, a   1 . Selanjutnya dengan menggunakan Teorema D.1

kita dapat simpulkan p | b .

Contoh
Banyak bilangan prima antara 10 dan 99 yang tetap merupakan bilangan prima jika
kedua digitnya dipertukarkan adalah ....
a. 9 d. 12
b. 10 e. 13
c. 11
(OSP 2007)
Jawaban:
Agar jika dibalik masih merupakan bilangan prima, maka digit puluhannya harus ganjil
dan bukan 5. Berikut ini adalah pasangan bilangan prima tersebut dengan hasilnya jika
kedua digit ditukar: 11 jika dibalik dirinya sendiri, 13,31 keduanya prima, (17,71)

keduanya prima, (19,91) tidak memenuhi karena 91  7 13 bukan prima, dan (37,73)
keduanya prima. Jadi ada 7 bilangan prima yang memenuhi.

Faktorisasi Prima
Setiap bilangan asli n  1 dapat dinyatakan secara tunggal sebagai

18
h

n  p1a1 p2a2 ... pkak

dengan k adalah suatu bilangan asli dan p1  p2  ...  pk bilangan-bilangan prima dan

ai  1 untuk setiap i  1, 2,..., k .

Faktorisasi prima ini, akan memudahkan kita dalam menganalisa suatu bilangan bulat.
Perhatikan bahwa jika bentuk faktorisasi prima dari n adalah
n  p1a1 p2a2 ... pkak ,

maka kita mengetahui beberapa hal sebagai berikut:


1. n mempunyai k faktor prima yang berbeda yaitu p1 , p2 ,..., pk .

2. Banyak faktor positif dari n adalah 1  a1 1  a2  ... 1  ak  .

Contoh
Faktorisasi prima dari 5220 adalah…
a. 22.32.145 c. 22.32.5.29 e. 22.35.5
b. 22.33.5.9 d. 24.3.5.7
(OSK 2003)
Jawaban:
Dengan menggunakan pohon faktor, mudah dipahami bahwa 5220  22.32.5.29 . (C)

Contoh
Joko mengalikan tiga bilangan prima berbeda sekaligus. Ada berapa faktor berbeda dari
bilangan yang dihasilkan…
a. 3 c. 5 e. 8
b. 4 d. 6
(OSK 2004)
Jawaban:
Misalkan bilangan-bilangan prima yang ia pilih adalah p, q, r , maka hasil perkaliannya

adalah pqr  p1q1r1 . Dengan demikian, banyak faktornya adalah 1  11  11  1  8

faktor. (E).

19
h

Penggunaan lain dari faktorisasi prima adalah mencari FPB dan KPK dua bilangan
bulat. Jika kitas udah mendapatkan faktorisasi prima dari dua bilangan bulat, sebut saja
a dan b, maka pastilah faktorisasi dari keduanya dapat dinyatakan sebagai
a  p1s1 p2s2 ... pksk dan b  p1t1 p2t2 ... pktk

dengan pi prima dan si , ti  0 untuk setiap i  1, 2,..., k . Dari informasi ini, dengan
mudah dapat dipahami bahwa
gcd  a, b   p1m1 p2m2 ... pkmk dan  a, b   p1M1 p2M 2 ... pkM k

dengan mi  min ai , bi  dan M i  maks ai , bi  untuk setiap i  1, 2,..., k .

Contoh
Kelipatan persekutuan terkecil dari 210, 42, dan 70 adalah .....
a. 14 c. 420 e. 1260
b. 210 d. 7
(OSK 2003)
Jaawaban:
Karena 210  2.3.5.7 , 42  2.3.7 , dan 70  2.5.7 , maka  210, 42, 70  2.3.5.7  210 .

(B).

Selain itu, dengan faktorisasi prima ini kita dapat menurunkan teorema berikut ini.

Teorema E.5
Hasil kali dari sebarang n bilangan bulat berurutan habis dibagi n ! .

Contoh
Buktikan bahwa  n  1 n  n3  1 senantiasa habis dibagi oleh 6 untuk semua bilangan

asli n. (OSN 2003)


Jawaban:
Perhatikan bahwa  n  1 n  n3  1   n  1 n  n  1  n 2  n  1 . Karena  n  1 n  n3  1

memuat perkalian 3 bilangan bulat berurutan yaitu n  1, n, dan n  1 maka jelas bahwa

 n  1 n  n3  1 habis dibagi 3!  6 untuk setiap bilangan asli n.

20
h

F. Persamaan Diophantine

Dalam menyelesaikan persamaan dengan penyelesaian bilangan bulat (yang selanjutnya


kita sebut persamaan Diophantine) tentu akan lebih mudah karena kita hanya dibatasi
penyelesaian dalam bilangan bulat. Sebagai contoh jika kita akan mencari pasangan
bilangan real  x, y  yang memenuhi xy  2 , tentu akan ada tak hingga banyaknya yaitu

 2
semua pasangan bilangan real  x,  untuk setiap bilangan real tak nol x, pasti
 x
merupakan solusi xy  2 . Akan tetapi jika kita akan mencari pasangan bilangan bulat

 x, y  yang memenuhi persamaan xy  2 , maka solusinya hanya ada 4 yaitu

1, 2  ,  2,1 ,  1, 2  , dan  2, 1 . Mengapa demikian? Untuk lebih jelasnya simak

uraian berikut ini.

Persamaan Diophantine Linear

Persamaan ini adalah persamaan yang paling sederhana, karena kita bisa langsung
mencari solusi umumnya. Bentuk umum dari persamaan ini adalah ax  by  c dengan
a, b, c  .

Teorema F.1.
Persamaan Diophantine linier ax  by  c mempunyai solusi jika dan hanya jika

gcd  a, b  | c

Teorema F.2.
Diketahui persamaan Diophantine ax  by  c . Jika  x0 , y0  adalah salah satu solusinya

maka solusi umum dari persamaan tersebut adalah


b b a
x  x0  k dan x  x0  k x  y0  k .
gcd  a, b  gcd  a, b  gcd  a, b 

21
h

Contoh
Bilangan 43 dapat dinyatakan ke dalam bentuk 5a  11b , karena untuk a  13 dan b  2 ,
nilai dari 5a  11b adalah 43. Manakah dari tiga bilangan 37, 254, dan 1986 yang tidak
dapat dinyatakan dalam bentuk 5a  11b ?
a. 1983 c. 254 dan 1986 e. tak ada
b. 254 d. semua
(OSK 2005)
Jawaban:
Perhatikan bahwa gcd  5,11  1 , sehingga 5a  11b  n akan punya solusi jika dan

hanya jika 1| n . Akan tetapi karena 1| n maka untuk setiap bilangan bulat n kita selalu
dapat menemukan a dan b sehingga 5a  11b  n . Jadi 37, 254, dan 1986 semuanya
dapat dinyatakan dalam bentuk 5a  11b . (E).

Contoh
16 dapat dinyatakan sebagai 3x  7 y sebab jika x diganti dengan 3 dan y diganti dengan
1 diperoleh 3.3+7.1 yang bernilai 16. Tujuh bilangan antara 100 dan 122 yang dapat
dinyatakan ke dalam bentuk 6 x  9 y adalah … (OSP 2005)
Jawaban:
gcd  6,9   3 , sehingga 6 x  9 y  n akan punya solusi jika 3| n . Jadi tujuh bilangan

yang dimaksud adalah semua n dengan 3| n yang terletak antara 100 dan 122 yakni
102, 105, 108, 111, 114, 117, dan 120. .

Persamaan Diophantine tak Linier


Persamaan ini sangat banyak bentuknya, kita tidak mungkin mengkarakteristik satu
persatu. Di sini kita hanya memaparkan beberapa teknik melalui contoh-contoh soal,
termasuk untuk pertidaksamaan yang penyelesaiannya bilangan bulat.

Contoh (teknik temfaktoran)


1 1 1 1 1
Misalkan m dan n adalah bilangan bulat dan 0  m  n . Jika   maka  
m n 3 m n
...

22
h

2 1 5
a. c.  e.
3 6 6
1 2
b. d. 
6 3
(OSK 2006)
Jawaban:
1 1 1 mn 1
     mn  3m  3n  mn  3m  3n  9  9   m  3 n  3   9 .
m n 3 mn 3
Karena 0  m  n maka m  3  1  m  4 dan n  3  9  n  12 , sehingga
1 1 1 1 3 1 2 1
      . (B).
m n 4 12 12 12 6

Contoh (tenik pemfaktoran dan keterbagian)


Ada berapa banyakkah pasangan terurut bilangan asli  a, b  dengan syarat a  b , dan

FPB  a, b   4 serta KPK  a, b   140 ? (OSP 2005)

Jawaban:
Dari hubungan antara FPB dan KPK kita punya
ab ab
KPK  a, b    140   ab  4 140  ab  16  5  7
FPB  a, b  4

tentu saja dengan syarat a dan b keduanya habis dibagi 4 karena FPB  a, b   4 . Dengan

demikian a  4m dan b  4n dengan mn  35 , tetapi karena a  b maka m  n .


Banyak pasangan bilangan asli  m, n  yang memenuhi mn  35 dan m  n adalah 2

yaitu 1,35  , dan  5, 7  . Jadi ada 2 pasangan bilangan asli  a, b  dengan syarat a  b ,

dan FPB  a, b   4 serta KPK  a, b   140 .

Contoh (teknik pembatasan)


Diketahui bentuk x 2  3 y 2  n , dengan x dan y adalah bilangan-bilangan bulat.
a. Jika n  20 , bilangan berapa sajakah n tersebut, dan diperoleh dari pasangan
(x,y) apa saja?
b. Tunjukkan bahwa tidak mungkin menghasilkan x 2  3 y 2  8 .

23
h

(OSN 2005)
Jawaban:
a. Nilai x 2 yang mungkin adalah 0,1, 4,9,16 dan nilai 3y 2 yang mungkin adalah
0,3,12 . Dengan demikian, nilai n  20 yang mungkin adalah 0 diperoleh dari

pasangan  0, 0  , 1 diperoleh dari pasangan  1, 0  , 4 diperoleh dari pasangan

 2, 0  dan  1, 1 , 9 diperoleh dari pasangan  3, 0  , 16 diperoleh dari

pasangan  4, 0  dan  2, 2  , 3 diperoleh dari pasangan  0, 1 , 12 diperoleh

dari pasangan  0, 2  dan  3, 1 , 13 diperoleh dari pasangan  1, 2  , 7

diperoleh dari pasangan  2, 1 , dan 19 diperoleh dari pasangan  4, 1 .

b. Dari uraian (a) jelas bahwa x 2  3 y 2  8 tidak punya solusi karena x 2  3 y 2


hanya akan menghasilkan 0, 1, 3, 4, 7, 9, 12, 13, 16, dan 19.

24
Peluang dan Statistika

Secara garis besar peluang adalah ilmu yang mempelajari perkiraaan berdasar kejadian-
kejadian tertentu. Sedangkan statistika adalah ilmu yang mempelajari tentang pengumpulan,
pengolahan dan penyajian data.

A. Kaidah Pencacahan
Penghitungan (counting) yang akan kita pelajari pada sub bab ini adalah kaidah penjumlahan,
kaidah perkalian, permutasi dan kombinasi.

Kaidah penjumlahan
Misalkan kita mempunyai suatu pekerjaan yang dapat dilakukan dengan k1 cara pertama,

atau k 2 cara ke-2, atau k 3 cara ke-3, ..., atau k n cara ke-n, maka banyak semua cara yang
dapat dilakukan adalah
k1  k2  k3  ...  kn cara
Perhatikan bahwa kata penghubung yang biasa dipakai adalah atau.

Contoh
s
Pecahan adalah pecahan sejati, jika s  t , dan faktor persekutuan terbesarnya adalah 1.
t
Jika t memiliki nilai mulai dari 2 sampai dengan 9, dan s bilangan positif, maka banyaknya
pecahan sejati berbeda yang dapat dibuat adalah ......
a. 26 c. 28 e. 36
b. 27 d. 30
(OSK 2004)
Jawaban:
Jika t  2 maka s  1 ada 1 pecahan,
jika t  3 maka s  1, 2 ada 2 pecahan,
jika t  4 maka s  1,3 ada 2 pecahan,
jika t  5 maka s  1, 2,3, 4 ada 4 pecahan,
jika t  6 maka s  1,5 ada 2 pecahan,
jika t  7 maka s  1, 2,3, 4,5,6 ada 6 pecahan,
jika t  8 maka s  1,3,5,7 ada 4 pecahan, dan
jika t  9 maka s  1, 2, 4,5,7,8 ada 6 pecahan.
Dengan demikian, menurut kaidah penjumlahan, banyak pecahan sejati yang dapat dibuat
adalah 1  2  2  4  2  6  4  6  27 pecahan. (B).

Kaidah Perkalian
Misalkan kita mempunyai suatu pekerjaan yang harus dilakukan dengan n-langkah yakni
ada k1 cara untuk langkah pertama, k 2 cara untuk langkah ke-2, k 3 cara untuk langkah ke-3,

..., dan k n cara untuk langkah ke-n, maka banyak semua cara yang dapat dilakukan adalah

k1  k2  k3  ...  kn cara
Perhatikan bahwa kata penghubung yang biasa dipakai adalah dan.

Contoh
Misalkan A  1, 2,3 dan B  a, b, c . Banyaknya korespondensi satu-satu yang dapat

dibuat dari A ke B adalah ...


a. 1 c. 6 e. 27
b. 3 d. 9
(OSK 2006)
Jawaban:
Kita akan memilih peta dari 1, 2, dan 3. Peta dari 1 ada 3 kemungkinan (a,b, dan c), peta dari
2 ada 2 kemungkinan (satu elemen di B telah menjadi peta dari 1), dan peta dari 3 tinggal 1
kemungkinan. Dengan demikian, banyak korespondensi satu-satu dari A ke B adalah
3 2 1  6 . (C). 3 2 1  6

Permutasi
Permutasi adalah penyusunan kembali suatu objek-objek tertentu, permutasi r dari n ditulis
dengan n Pr adalah banyak cara mengambil r unsur dari n objek dengan memperhatikan
urutan. Yang dimaksud dengan memperhatikan urutan adalah bahwa dua unsur yang sama
tetapi urutannya berbeda dianggap berbeda, sebagai contoh susunan ab dianggap berbeda
denga ba. Perhatikan bahwa untuk memilih r dari n unsur, berarti kita membutuhkan r
tempat.

Teorema A.1
n!
Pr  , di mana n !  1.2.3...  n  1 n dan 0!  1.
n
 n  r !
Bukti:
Tempat pertama kemungkinan diisi oleh n objek (ingat bahwa kita memilih di antara n
objek), tempat kedua kemungkinan diisi oleh n  1 (satu objek telah dipakai pada tempat
pertama), tempat ketiga kemungkinan diisi oleh n  2 , dan seterusnya sampai tempat ke r
kemungkinan diisi oleh n   r  1 . Dengan demikian, menggunakan kaidah perkalian kita

peroleh

n Pr  n  n  1 n  2  ...  n   r  1 

 n  n  1 n  2  ...  n   r  1 
 n  r  n  r  1 ...2.1 .
 n  r  n  r  1 ...2.1
n!
 .
 n  r !

Pada uraian di atas, dianggap bahwa setiap unsurnya berbeda. Jika kita punya n objek yang
terdiri dari k1 unsur yang sama, k 2 unsur yang sama, ..., km unsur yang sama, maka untuk

setiap susunan yang memuat objek ke-i yang sama akan dihitung sebanyaj ki ! , sehingga
banyak susunannya adalah
n!
n Pk1 ,k2 ,...,km 
k1 !k2 !...km !
Satu lagi macam permutasi adalah permutasi siklis, yaitu banyak cara menyusun n objek
pada lingkaran. Pada permutasi ini, kita ’mengikat’ dua unsur (supaya tidak ada
pengulangan) kemudian kita pandang sebagai permutasi dari n  1 unsur yang berbeda,
sehingga
P  n siklis    n  1!

Kombinasi
n
Kombinasi r dari n ditulis dengan   adalah banyak cara mengambil r unsur dari n objek
r 
tanpa memperhatikan urutan. Yang dimaksud dengan tanpa memperhatikan urutan
susunan ab dianggap sama denga ba.

Teorema A.2
n n!
 
 r   n  r  !r !
Bukti:
Perhatikan bahwa kita memilih r dari n objek yang dapat dilakukan dengan n Pr cara. Akan
tetapi pada perhitungan ini urutan masih diperhatikan, untuk setiap pilihan r unsur pasti akan
diulang sebanyak r !, sehingga untuk menghilangkan urutan maka banyak cara ini kita bagi
dengan r !. Dengan demikian
 n  n Pr n!
   .
 r  r !  n  r  !r !

Contoh
Enam belas tim sepak bola mengikuti turnamen. Pertama-tama mereka dikelompokkan ke
dalam 4 kelompok dengan masing-masing 4 tim di setiap kelompoknya. Di setiap kelompok
mereka saling bermain satu sama lain satu kali. Dua tim yang memiliki peringkat teratas
selanjutnya maju babak berikutnya yang menggunakan sistem gugur (kalah langsung
tereliminasi) sampai ditemukan juaranya. Berapa banyak pertandingan yang berlangsung
dalam turnamen tersebut? (OSP 2004)
Jawaban:
 4
Banyak pertandingan pada tiap grup adalah    6 , sehingga banyak pertandingan pada
 2
sistem grup adalah 4  6  24 pertandingan. Banyak tim yang maju ke babak berikutnya ada
sebanyak 8 tim, di mana akan dipakai sistem gugur. Perhatikan bahwa dalam sistem gugur,
setiap pertandingan menggugurkan 1 tim. Juara akan ditemukan jika 7 tim yang lainnya
gugur dan ini membutuhkan 7 pertandingan. Dengan demikian, banyak pertandingan yang
berlangsung dalam turnamen tersebut adalah 24  7  31 pertandingan.

Contoh (penggunaan dalam geometri)


Perhatikan segi enam berikut. Banyak segitiga yang dapat ditemukan pada gambar tersebut
adalah .....

(OSP 2005)
Jawaban:
Segienam akan diperoleh dari sebarang 3 titik segienam tersebut, sehingga banyak
6
segitiganya adalah    20 segitiga.
3

Salah satu penggunaan dari kombinasi ini adalah menghitung suku-suku pada penjabaran
pangkat suku dua yang selanjutnya disebut Binomial Newton. Selengkapnya adalah sebagai
berikut:

n  n  n n  n  n 1  n  n
 a  b    a n    a n 1b    a n 2b 2  ...    a n k b k  ...    ab    b .
n

0 1   2 k  n  1  n

Contoh
8
 2
Konstanta dari  3x3   adalah…
 x
a. 14.328 c. 16.112 e. 17.128
b. 15.552 d. 16.128
(OSK 2007)
Jawaban:
Dengan menggunakan Binomial Newton, setiap sukunya berbentuk

 8  3 8 i  2   8  8i  8  8i 8


i i
3 8 i  1 
   3x        3  2   x       3  2  x x    38i  2  x 24 4i .
i i 24 3i  i i

i   x  i   x  i  i 
Konstanta diperoleh jika pangkat dari x adalah 0, yakni terjadi jika 24  4i  0  i  6 .
Dengan demikian konstantanya adalah
 8  8 6 8  7  6! 2
  3  2    3  64  28  9  64  16.128 .
6

6 2  6!
Jawaban (D).

Di samping itu, kombinasi juga digunakan untuk menghitung banyak lintasan terpendek pada
suatu grid. Perhatikan grid berukuran m  n berikut ini.

Kita akan menghitung banyak lintasan terpendek dari A ke B. Lintasa terpendek hanya
terjadi jika kita selalu berjalan ke kanan dan ke atas. Dengan demikian, kita harus
melangkah sejauh m  n dengan m diantaranya ke kanan dan sisanya ke atas, sehingga
kita memilih m langkah ke kanan dari m  n langkah kemudian memilih n langkah untuk
 m  n  n   m  n 
ke atas dari sisanya, ini ada sebanyak      .
 m  n   m 
m  n
Jadi, banyak lintasan terpendek dari A ke B adalah  .
 m 

Contoh
Banyak jalan terpendek dari P ke Q adalah ….
a. 4 d. 60
b. 16 e. 80
c. 22
(OSP)
Jawaban:
Tempatkan titik-titik A, B, C, D, E dan F seperti gambar berikut ini.

Perhatikan bahwa setiap lintasan terpendek dari P ke Q pasti melewati A, B, C, atau D.


Keempat rute PAQ, PBQ, PCQ, dan PDQ tidak mungkin beririsan, sehingga banyak rute
terpendek dari P ke Q kita tulis # PQ adalah
# PQ  # PAQ  # PBQ  # PCQ  # PDQ
 4  5 
 1.1      # PECQ  # PFCQ  # PFDQ
 3 1 
 3   3   3  3  3   3 
 1  4.5   1         1.1
 2   2  1 1  2   1 
 1  20  3.3  3.3.3  3
 60
Jadi banyak rute terpendek dari P ke Q adalah 60 rute. (D)

B. Peluang

Pada dasarnya, ilmu peluang adalah ilmu perkiraan. Artinya seberapa mungkinkah suatu
kejadian dapat terjadi. Sebelum membahas definisinya, kita akan melihat terlebih dahulu apa
itu ruang sampel dan titik sampel. Misalkan kita melemparkan sebuah dada muka 6, maka
kemungkinan muka dadu yang keluar adalah 1, 2, 3, 4, 5, atau 6. Nah, himpunan semua
kemungkinan ini yang disebut ruang sampel. Jadi, ruang sampel suatu kejadian adalah
himpunan dari semua kemungkinan yang mungkin dari kejadian tersebut. Sedangkan, titik
sampel adalah anggota dari ruang sampel.

Definisi B.1
Misalkan S adalah suatu ruang sampel dan A adalah suatu kejadian pada S. Jika n  A  dan

n  S  berturut-turut menyatakan banyak cara kejadian A terjadi dan banyak anggota ruang

sampel S, maka peluang kejadian A ditulis P  A  dan didefinisikan dengan

n  A
P  A  .
nS 

Nilai peluang berkisar antara 0 sampai 1, jika nilai peluangnya 0 disebut kemustahilan dan
jika nilai peluangnya 1 disebut kepastian.

Contoh
Indonesia akan mengirim delegasi Olimpiade Sains Internasional (OSI) tingkat SMP pada
tahun 2006. Delegasi ini terdiri atas tiga siswa SMP yang harus dipilih secara acak dari 10
kandidat yaitu enam siswa bidang Sains dan empat siswa bidang Matematika. Berapa
peluang terpilihnya delegasi OSI yang terdiri 2 siswa dari bidang Sains dan 1 siswa dari
bidang Matematika? (OSP 2006)
Jawaban:
10 
Banyak ruang sampelnya adalah banyak cara memilih 3 dari 10 siswa yaitu    120 cara.
3
Banyak cara memilih delegasi 2 siswa sains dan 1 matematika berturut-turut adalah banyak
 6  4
cara memilih 2 dari 6 siswa dan 1 dari 4 siswa yaitu sebanyak   .    15.4  60 . Dengan
 2  1 
demikian peluang terpilihnya delegasi OSI yang terdiri 2 siswa dari bidang Sains dan 1 siswa
60 1
dari bidang Matematika adalah  .
120 2

Sekarang perhatikan diagram Venn berikut ini

Dari teori himpunan kita punya n  A  B   n  A   n  B   n  A  B  , dengan membagi

kedua ruas dengan n  S  akan kita peroleh

n  A  B  n  A n  B  n  A  B 
    P  A  B   P  A  P  B   P  A  B  .
nS  nS  nS  n S 

Jadi, rumus kejadian majemuk dua kejadian adalah


P  A  B   P  A  P  B   P  A  B 

(keterangan: P  A  B  menyatakan peluang kejadian A atau B, dan P  A  B  menyatakan

peluang kejadian A dan B).

Teorema B.1
Khusus jika B  Ac maka A  Ac  S dan A  Ac   , sehingga
1  P  S   P  A   P  Ac   P  A c   1  P  A 

Di samping itu, kita juga definisikan


 A dan B saling lepas jika P  A  B   P  A   P  B  atau dengan kata lain A  B  

,
 A dan B saling bebas jika P  A  B   P  A   P  B  .

Tambahan: Jangan lupakan hukum D’Morgan


 A  B  Ac  B c dan  A  B   Ac  B c
c c

Contoh
Dua mata uang dilempar empat kali berturut-turut. Peluang muncul angka pertama kali pada
pelemparan keempat adalah ….
1 3 1
a. c. e.
44 44 4
2 1
b. d.
44 42
(OSK 2007)
Jawaban:
Pertama akan kita hitung peluang muncul angka pada setiap lemparan. Jika A adalah
kejadian muncul angka, maka Ac adalah kejadian tidak muncul angka atau dengan kata lain
kedua mata uang yang dilempar keduanya kuncul gambar (GG), karena ada anggota ruang

sampel adalah 4 maka P  Ac   , sehingga P  A   1  P  Ac   . Sekarang perhatikan


1 3
4 4
bahwa agar angka empat muncul pertama kali pada lemparan keempat haruslah pada
pelemparan pertama, kedua, dan ketiga tidak muncul angka yang masing-masing peluangnya
1 3
adalah , sedangkan pelemparan keempat muncul angka yang peluangnya adalah .
4 4
Dengan demikian,. Peluang muncul angka pertama kali pada pelemparan keempat adalah
1 1 1 3 3
. . .  . (C)
4 4 4 4 44

Contoh
Satu set soal terdiri dari 3 soal dengan pilihan jawaban Benar (B) atau Salah (S), serta 3 soal
pilihan ganda dengan jawaban A, B, C, atau D. Seseorang menjawab semua soal secara acak.
Berapa peluang ia hanya benar 2 soal? (OSN 2006)
Jawaban:
Untuk tipe soal benar salah kita sebut tipe BS, tiap soal ada 2 kemungkinan yaitu benar atau
salah, sehingga banyak kemungkinan jawaban untuk semua tipe soal ini ada sebanyak
2.2.2  8 .
Untuk soal pilihan ganda kita sebut tipe PG, tiap soal ada 4 kemungkinan jawaban, sehingga
banyak kemungkinan jawaban untuk semua soal tipe ini ada sebanyak 4.4.4  64 .
Perhatikan bahwa jawaban seseorang hanya benar 2 soal terjadi jika: 2 soal BS dijawab benar
dan 3 soal PG salah semua kita sebut kejadian X, 1 soal BS dan 1 soal PG masing-masing
dijawab benar kita sebut kejadian Y, serta 2 soal PG dijawab benar dan 3 soal BS salah semua
 3   3  3  3
       
kita sebut kejadian Z. Kita punya P  X     .    , P Y     .   
2 3 3 1 1 9
, dan
8 64 512 8 64 512
 3  3 
   
PZ     .  
3 2 3
, sehigga karena X, Y, dan Z saling lepas maka peluang ia hanya
8 64 512
3 9 3 15
menjawab benar 2 soal adalah P  X   P Y   P  Z      .
512 512 512 512

C. Ukuran pemusatan data


Pada sekumpulan data tertentu terkadang kita cukup melihat pemusatan dari data tersebut
yang dapat kita anggap ”mewakili” semua data tersebut. Pemusatan data ini disebut dengan
tiga tendensi sentral yang terdiri dari mean, median, dan modus.

Mean atau dalam istilah Indonesia kita kenal dengan rata-rata atau rataan dan kita

lambangkan dengan x , dan didefinisikan dengan hasil bagi dari jumlah semua data dengan
banyak datanya, dengan kata lain:
x1  x2  x3  ...  xn
jika diketahui data x1 , x2 , x3 ,..., xn , maka rata-ratanya adalah x  .
n
Median dilambangkan me adalah data tengah setelah diurutkan.

Modus dilambangkan dengan mo adalah nilai yang paling sering muncul.

Contoh
Rata-rata sembilan bilangan adalah 6. Satu diantara kesembilan bilangan dibuang. Rata-rata
delapan bilangan yang tinggal adalah 6 ½ . Bilangan yang dibuang adalah ...... (OSK 2004)
Jawaban:
Misalkan bilangan yang dibuang adalah x . Perhatikan bahwa jumlah semua data semula
adalah 9  6  54 . Jika x dibuang maka jumlahnya menjadi 54  x dengan banyak data 8.
Dengan demikian,
54  x 1
 6  54  x  52  x  2 .
8 2
Jadi bilangan yang dibuang adalah 2.

Contoh
Tiga bilangan bulat membentuk kumpulan data yang berata-rata 10. Banyaknya kombinasi
bilangan yang (sebutkan pula datanya), jika diketahui selisih data terbesar dan terkecilnya
tidak lebih dari 4 adalah ..... (OSP 2004)
Jawaban:
Misalkan 3 bilangan tersebut adalah x, y, dan z dengan x  y  z . Dari informasi yang
x yz
diberikan kita tahu  10  x  y  z  30 dan z  x  4 .
3
Untuk sebarang 0  a  4 akan kita cara semua kombinasi data yang mungkin jika z  x  a .
Perhatikan bahwa z  x  a sehingga 30  x  y  z  x  y  x  a  2x  y  a  3x  a  3x .
Dari sini kita punya x  10. Jelas bahwa z  10 , sedangkan di lain pihak
z  x  a  10  a  14 , sehingga 10  z  14 .
 Untuk z  10 , diperoleh x  10 dan y  10 ,

 Untuk z  11, diperoleh pasangan  x, y  :  8,11 dan  9,10  ,

 Untuk z  12 , diperoleh pasangan  x, y  :  8,10  dan  9,9  ,

 Untuk z  13 , tidak ada pasangan  x, y  yang memenuhi karena x  z  a  9 tetapi

17  x  y  2 x  x  8 yang jelas tidak mungkin terjadin secara bersamaan.

Jadi ada sebanyak 5 kombinasi bilangan  x, y , z yang mungkin yaitu 8,10,12 , 8,11,11 ,

9,9,12 , 9,10,11 , dan 10,10,10 .


h

Pengayaan

Materi ini adalah materi tambahan bagi yang mau belajar olimpiade matematika lebih
lanjut. Materi ini dapat dikatakan tidak termasuk dalam silabus OSN, namun materi ini
akan berguna untuk mengerjakan soal olimpiade nasional dari negara-negara lain. Di
samping itu, materi ini merupakan materi wajib untuk olimpiade matematika tingkat
SMA.

A. Prinsip Induksi Matematika

Pada dasarnya, induksi matematika adalah suatu “alat” yang digunakan untuk
membuktikan suatu pernyataan benar atau berlaku pada suatu himpunan. Biasanya
himpunan yang dimaksud adalah himpunan bilangan asli atau dapat juga merupakan
bilangan bulat. Sebagai contoh pernyataan “untuk setiap bilangan asli n berlaku
n  n  1
1  2  ...  n  ” ini adalah suatu pernyataan yang pada himpunan bilangan asli,
2
n  n  1
artinya jika 1  2  ...  n  kita tulis dengan P  n  maka pernyataan tersebut
2
dapat kita tulis “untuk setiap bilangan asli berlaku P  n  ”. Nah, pernyataan di atas

adalah salah satu contoh yang nantinya akan kita buktikan dengan menggunakan prinsip
induksi matematika atau cukup kita sebut induksi matematika.

Perhatikan gambar sebagai berikut:

Misalkan kita punya batu bata yang kita susun seperti gambar di atas sehingga h  x .
Sekarang batu bata paling kiri kita „pukul‟ ke arah kanan sampai roboh. Apa yang
terjadi? Batu bata yang berada tepat di sebelah kanannya akan roboh juga, kemudian ini

Nanang S

1
h

mengakibatkan yang di sebelah kanannya roboh juga dan seterusnya sehingga kita
dapatkan bahwa semua batu bata roboh.

Mari kita lihat secara detail proses di atas:


Diketahui (i). batu bata pertama roboh, dan (ii). jika setiap batu bata yang roboh
mengakibatkan batu bata yang berada tepat di sebelah kanannya juga roboh. Dua
informasi tersebut cukup bagi kita untuk menyimpulkan bahwa semua batu bata roboh.
Induksi matematika mengadopsi/meniru proses di atas. Secara formal, induksi
matematika didefinisikan sebagai berikut:

Induksi Matematika
Diberikan pernyataan P  n  untuk setiap bilangan asli n. Jika diketahui

(i). P 1 merupakan pernyataan yang benar,

(ii). Untuk setiap bilangan asli n, jika P  n  benar berakibat P  n  1 juga benar,

maka pernyataan P  n  merupakan pernyataan yang benar untuk setiap bilangan asli n.

Untuk mempermudah, mari kita lihat bagaimana menggunakan induksi matematika


dalam membuktikan suatu pernyataan.

Langkah Induksi
Secara implisit untuk membuktikan suatu pernyataan P  n  benar untuk setiap bilangan

asli n, ada 3 langkah:


(i). membuktikan P 1 benar (disebut basis induksi),

(ii). mengasumsikan pernyataan benar untuk n  k (disebut asumsi induksi),


(iii). membuktikan pernyataan benar juga untuk n  k  1 (tentu dengan menggunakan
asumsi induksi pada langkah (ii)).

Nanang S

2
h

Mari kita lihat contoh penggunaannya.

Contoh
n  n  1
Buktikan bahwa untuk setiap bilangan asli n berlaku 1  2  ...  n  .
2
Jawaban:
Akan kita buktikan dengan induksi matematika.
11  1
(i). untuk n  1 , ruas kiri  1 , ruas kanan   1  ruas kiri. Pernyataan benar
2
untuk n  1 .
k  k  1
(ii). asumsikan pernyataan benar untuk n  k , artinya 1  2  ...  k  .
2
(iii). akan dibuktikan benar juga untuk n  k 1 . Perhatikan bahwa

1  2  ...  k   k  1 
k  k  1
  k  1 
 k  1 k  2   k  1   k  1  1 , yang
 
2 2 2
artinya bahwa pernyataan benar untuk n  k  1 .

Contoh
A adalah suatu himpunan bilangan. Himpunan A memiliki sifat tertutup terhadap
pengurangan, artinya hasil pengurangan dua bilangan di A akan menghasilkan bilangan
di A juga. Jika diketahui dua anggota A adalah 4 dan 9, tunjukkan bahwa:
a.0 ∈A,
b.-13∈A,
c. 74∈A.
Selanjutnya, daftarlah semua anggota himpunan A. (OSN 2005, Jakarta)
Jawaban:
a. Karena 4∈A maka dengan meggunakan sifat ketertutupannya terhadap
pengurangan kita punya 4  4  A  0  A,
b. 0, 4,9  A maka 0  4  4  A , sehingga 4  9  13  A,

Nanang S

3
h

c. 9, 13  A maka 9   13  22  A , 22   13  35  A , 35   13  48  A ,

48   13  61 A sehingga 61   13  74  A .

Sekarang akan kita datar semua anggota himpunan A. Jelas bahwa anggota-anggota A
merupakan bilangan bulat. Sekarang, akan kita buktikan bahwa semua bilangan bulat
merupakan anggota A. Dari soal bagian a, kita telah berhasil membuktikan bahwa
0  A , sehingga jika a  A maka 0  a  a  A , artinya kita cukup membuktikan
bahwa semua bilangan asli merupakan anggota A karena dari sifat ini lawannya
(negatifnya) juga merupakan anggota A. Perhatikan bahwa 9  4  5  A , sehingga
5  4  1 A , akibatnya 1 A . Sekarang akan kita gunakan induksi matematika untuk
mebuktikan bahwa setiap bilangan asli merupakan anggota A.
(i). untuk n  1 jelas benar karena 1 A ,
(ii). dianggap benar untuk n  k , artinya k  A ,
(iii). akan dibuktikan benar juga untuk n  k  1. Perhatikan bahwa k , 1 A , sehingga

k   1  k  1 A .

Ini membuktikan bahwa semua bilangan asli merupakan anggota A, selanjutnya kita
akan punya bahwa setiap lawan dari bilangan asli ini juga anggota A, selain itu 0 juga
anggota A. Jadi A adalah himpunan semua bilangan bulat atau dengan kata lain
A  ..., 3, 2, 1, 0,1, 2,3,... .

Induksi matematika yang telah kita bahas tadi adalah induksi matematika biasa. Selain
induksi matematika biasa, terdapat juga induksi matematika kuat atau cukup dikatakan
induksi kuat. Induksi ini lebih rumit dibandingkan dengan induksi biasa, sehingga ini
biasanya digunakan pada kompetisi internasional, sedangkan pada kompetisi nasional
biasanya hanya sampe induksi biasa. Perbedaan antara induksi kuat dan biasa terletak
pada basis induksi dan asumsinya. Basis induksi pada induksi kuat, tidak harus 1 dan
asumsi induksi yang digunakan lebih kuat daripada pada induksi biasa. Langkah
induksinya adalah sebagai berikut.

Nanang S

4
h

Langkah Induksi Kuat


Untuk membuktikan suatu pernyataan P  n  merupakan pernyataan yang benar untuk

setiap bilangan asli n  n0 . ada 3 langkah:

(i). membuktikan P  n0  benar (basis induksi),

(ii). mengasumsikan pernyataan benar untuk semua n dengan n0  n  k (asumsi

induksi),
(iii). membuktikan pernyataan benar juga untuk n  k  1 (tentu dengan menggunakan
asumsi induksi pada langkah (ii)).

Contoh
Buktikan bahwa untuk setiap bilangan asli n bentuk
n n
 1 5   1 5 
    
 2   2 
merupakan bilangan bulat.
Jawaban:
1 1
 1 5   1 5 
Untuk n  1 , kita punya       1 jelas merupakan bilangan bulat.
 2   2 
n n
 1 5   1 5 
Asumsikan benar untuk n  1, 2,..., k , artinya      merupakan
 2   2 
 1 5   1 5 
bilangan bulat untuk setiap n  1, 2,..., k . Kita tulis x    , y    .
 2   2 
Perhatikan bahwa
k 1 k 1
 1 5  1 5 
      x k 1  y k 1
 2   2 
  x  y   x k  y k   xy  x k 1  y k 1 
 1 x k  y k    2   x k 1  y k 1 
  x k  y k   2  x k 1  y k 1 

Nanang S

5
h

Karena menurut asumsi induksi x k  y k dan x k 1  y k 1 merupakan bilangan bulat maka


bentuk di atas juga merupakan bilangan bulat.

B. Kongruensi dalam Bilangan Bulat

Kongruensi merupakan cara lain untuk menyajikan keterbagian. Keterbagian yang


dibahas pada bab teori bilangan merupakan kejadian khusus dari kongruensi ini. Di sini
akan dibahas defnisi kongruensi dan sifat-sifatnya beserta beberapa contoh soal yang
berhubungan dengan kongruensi. Seperti halnya pada keterbagian, kongruensi
berhubungan dengan suatu bilangan bulat tertentu sebu saja n. yang nantinya akan
disebut dengan modulo n. Notasi dari kongruensi adalah '  ' dan dibaca kongruen.

Sebagai contoh a  b  mod n  dibaca: a kongruen dengan b modulo n. Sering kali, untuk
menyingkat penulisan, tanda kurung biasanya tidak ditulis. Dalam hal a tidak kongruen

dengan b kita tulis dengan a  b  mod n  .

Definisi
Diberikan bilangas asli n  1 . Untuk setiap bilangan a dan b didefinisikan a  b mod n

jika dan hanya jika n |  a  b  . Dengan kata lain, selsuai definisi keterbagian

a  b mod n jika terdapat bilangan bulat k sehingga a  kn  b .

Sebagai contoh 10  0mod5 , 2012  2mod10 , tetapi 5  3mod 4 , dan sebagainya.


Langsung dari definisi di atas, kita dapat turunkan beberapa sifat berikut ini.

Sifat-sifat Kongruensi
Diberikan sebarang bilangan asli m, n  1 . Untuk setiap a, b, c, d  berlaku
1. a  a mod n ,
2. Jika a  b mod n maka b  a mod n ,
3. Jika a  b mod n maka a  c  b  c mod n dan ac  bc mod n .
4. Jika a  b mod n dan c  d mod n maka ac  bd mod n .
5. Jika a  b mod n dan a  b mod m dengan gcd  m, n   1 maka a  b mod mn .

Nanang S

6
h

6. Jika a  b mod n dan a  b mod m maka a  b mod  m, n  .

Bukti hampir sama dengan bukti pada saat kita membahas keterbagian dan diserahkan
kepada pembaca sebagai latihan. Perhatikan sifat 4 di atas. Dari sifat tersebut diperoleh
akibat di bawah ini.

Akibat
Jika a  b mod n maka am  bm mod n untuk setiap bilangan bulat tak negatif m.
Bukti:
Jelas bahwa 1  1mod n , dan karena a  b mod n maka menurut sifat 4 kita punya
a2  b2 mod n , a3  b3 mod n , dan seterusnya. Dengan menggunakan prinsip induksi
matematika kita punya bahwa am  bm mod n untuk setiap bilangan bulat tak negatif m.

Kongruensi ini akan sangat berguna dalam menyelesaikan soal olimpiade.


Penggunaannya biasanya mulai dari hal yang sederhana sebagai contoh: bilangan
kuadrat selalu kongruen 0 atau 1 dalam modulo 3, bilangan kuadrat selalu kongruen 0
atau 1 dalam modulo 4, bilangan prima lebih dari 3 selalu kongruen 1 atau 5 modulo 6,
dan sebagainya. Mari kita lihat beberapa contoh penggunaanya.

Contoh
Ada berapa banyakkah bilangan asli yang tidak lebih besar dari 2004 yang bersisa 1
ketika dibagi 2, bersisa 2 ketika dibagi 3, bersisa 3 ketika dibagi 4, dan bersisa 4 ketika
dibagi 5? (OSN 2004)
Jawaban:
Soal ini pernah di bahas pada bab teori bilangan. Kita akan membahas dengan
menggunakan kongruensi. Misalkan n adalah bilangan yang memenuhi semua syarat di
atas. Bilangan n bersisa 1 ketika dibagi 2 artinya n  1  1mod 2 , bersisa 2 ketika
dibagi 3 artinya n  2  1mod3 , bersisa 3 ketika dibagi 4 artinya n  3  1mod 4 ,
dan bersisa 4 ketika dibagi 5 artinya n  4  1mod5 . Dengan demikian
n  1mod 2,3, 4, dan 5 yang artinta n  1mod  2,3, 4,5  n  1mod 60 , sehingga

bilangan yang memenuhi syarat di atas adalah bilangan yang berbentuk 60k  1 untuk

Nanang S

7
h

sebarang bilangan bulat k. Karena n merupakan bilangan asli maka jelas bahwa k  1 , di
25
samping itu ada syarat n  2004  60k  1  2004  60k  2005  k  33 , tetapi
60
karena k bulat maka k  33 . Jadi 1  k  33 , yang berarti ada sebanyak 33 bilangan asli
yang memenuhi.

Contoh
Tunjukkan bahwa tidak mungkin menghasilkan x 2  3 y 2  8 untuk setiap bilangan bulat
x dan y. (bagian dari soal OSN 2005)
Jawaban:
Pada bab teori bilangan, telah dibahas soal ini tetapi dengan teknik pembatasan. Di sini
kita akan menjawabnya dengan kongruensi. Perhatikan bahwa bilangan kuadrat selalu
kongruen dengan 0 atau 1 modulo 3, sehingga x2  0 atau 1 mod 3 dan 3 y 2  0mod3 ,

akibatnya 8  x 2  3 y 2  0 atau 1 mod 3. Hal ini tidaklah mungkin terjadi karena


8  2mod3 . Jadi tidaklah mungkin terdapat bilangan bulat x dan y yang memenuhi
x2  3 y 2  8. .

C. Segiempat Talibusur

Sesuai dengan namanya, segiempat talibusur adalah segiempat yang keempat sisinya
merupakan talibusur suatu lingkaran. Ada beberapa sifat istimewa dari segiempat
talibusur ini, mulai dari power point dari suatu titik di dalam atau di luar lingkaran,
Teorema Ptolemy, dan rumus luas segiempat talibusur.

Power Point titik di dalam lingkaran


Jika ABCD adalah segiempat talibusur dengan E merupakan titik potong kedua
diagonalnya maka EA  EC  EB  ED .

Nanang S

8
h

Bukti:
Perhatikan segitiga ADE dan BCE. Kita punya
E  E (bertolak belakang)
A  B (menghadap busur CD)
D  C (menghadap busur AB).
EA ED
Akibatnya EAD sebangun dengan EBC , sehingga  yang selanjutnya akan
EB EC
kita peroleh EA  EC  EB  ED .

Dengan cara yang sama, jika talibusur AB dan CD berpotongan di E, sedangkan ET


adalah garis singgung lingkaran maka
EB  EA  ED  EC  ET 2

Dua teorema di atas kita sebut dengan powe point suatu titik terhadap lingkaran.

Contoh

Nanang S

9
h

Dari titik sudut A ditarik garis yang memotong sisi BC di X dan lingkaran luar ABC di
1 1 4
Y. Buktikan bahwa   . (Baltic Way, 2004)
AX XY BC
Jawaban:
1 1 2
Dengan menggunakan GM  HM kita punya   ……………(1)
AX XY AX . XY
Karena X adalah titik potong talibusur BC dan AY maka dengan aturan power point di
1 1 2
titik X kita punya AX .XY  BX .XC , sehingga (1) akan menjadi   .
AX XY BX . XC
BX  XC BC
Terakhir dengan menggunakan GM  AM kita punya BX . XC   .
2 2
1 1 2 2 4
Dengan demikian, kita peroleh     . (terbukti).
AX XY BX . XC BC / 2 BC

Teorema Ptolemy
Jika ABCD adalah segiempat talibusur maka AC  BD  AB  CD  AD  BC .

Bukti:
Misalkan E adalah titik pada AC sehingga ADE  BDC .

Nanang S

10
h

Perhatikan bahwa
ADE  BDC
DAE  DBC (mengahadap busur DC)
DEA  DCB ,
AD AE
artinya DAE sebangun dengan DBE , sehingga  yang selanjutnya kita
BD BC
peroleh AD  BC  AE  BD …………………………………………………………..(1)
Sekarang, perhatikan bahwa
ADB  EDC
ABD  ECD (menghadap busur AD)
BAD  DEC
BD AB
artinya DBA sebangun dengan DCE , sehingga  yang selanjutnya kita
CD EC
peroleh AB  CD  EC  BD ………………………………………………………….(2)
Dengan menjumlahkan persamaan (1) dan (2) kita peroleh
AD  BC  AB  CD  AE  BD  EC  BD   AE  EC   BD  AC  BD .

Dan kita telah selesai membuktikan teorema Ptolemy.

Rumus luas segiempat talibusur (Bhrahmagupta)


Jika ABCD adalah segiempat talibusur dengan panjang sisi-sisi a, b, c, dan d maka luas
segiempat talibusur ABCD adalah

 ABCD    s  a  s  b  s  c  s  d 
abcd
dengan s  .
2
Bukti diserahkan kepada pembaca sebagi latihan. Perhatikan bahwa jika D berimpit
dengan C maka d  0 sehingga ABCD menjadi segitiga dan rumus di atas menjadi
rumus luas segitiga formula Heron.

Hal yang sangat menarik adalah bahwa beberapa teorema pada segiempat talibusur
dapat dibalik. Berikut ini ada beberapa pernyataan yang merupakan balikan/konvers
dari beberapa teorema di atas.

Nanang S

11
h

1. Jika pada segiempat ABCD berlaku DAB  DCB  1800 , maka ABCD adalah
segiempat talibusur.
2. Jika pada segiempat ABCD berlaku BAD  BCD , maka ABCD adalah
segiempat talibusur.
3. Jika kedua diagonal segiempat ABCD berpotongan di E dan EA  EB  EC  ED
maka ABCD adalah segiempat talibusur.

Contoh
Dari titik sudut A ditarik garis yang memotong sisi BC di X dan lingkaran luar ABC di
1 1 4
Y. Buktikan bahwa   . (Baltic Way, 2004)
AX XY BC
Jawaban:
1 1 2
Dengan menggunakan GM  HM kita punya   ……………(1)
AX XY AX . XY
Karena X adalah titik potong talibusur BC dan AY maka dengan aturan power point di
1 1 2
titik X kita punya AX .XY  BX .XC , sehingga (1) akan menjadi   .
AX XY BX . XC
BX  XC BC
Terakhir dengan menggunakan GM  AM kita punya BX . XC   .
2 2
1 1 2 2 4
Dengan demikian, kita peroleh     . (terbukti).
AX XY BX . XC BC / 2 BC

D. Pigeon Hole Principle (PHP) dan Pewarnaan

Nanang S

12
h

Dalam bahasa Indonesia PHP biasa disebut dengan prinsip kotak sarang merpati.
Penggunaan dari prinsip ini adalah untuk membuktikan suatu eksistensi. Sedangkan
pewarnaan biasanya berhubungan dengan pengubinan.

PHP (Pigeon Hole Principle)


Jika terdapat n  1 merpati yang akan menempati n sarang maka pasti terdapat sarang
yang ditempati 2 merpati atau lebih.

Sekilas prinsip ini terlihat trivial, tetapi ini sangat berguna untuk membuktikan
eksistensi sesuatu. Sebagai contoh pada sebarang grup yang berisi 13 orang pasti ada 2
orang yang bulan lahirnya sama. Prinsip di atas dapat kita perluas seperti yang
dijelaskan di bawah ini.

PHP extension
Jika terdapat an  1 merpati yang akan menempati n sarang maka pasti terdapat sarang
yang ditempati a  1 merpati atau lebih.
Contoh
Misalkan A adalah sebarang himpunan 20 bilangan bulat yang dipilih dari barisan
aritmetika 1,4,…,100. Buktikan bahwa pasti terdapat dua bilangan berbeda di A yang
jumlahnya 104. (Putnam 1978)
Jawaban:
Kita kelompokkan barisan aritmetika di atas ke dalam 19 kelompok
1 , 52 , 4,100 , 7,97 , 10,94 ,..., 49,55 .

Karena kita memilih 20 bilangan dan kita hanya punya 19 kelompok maka dengan PHP
pasti ada 2 bilangan yang terpilih dari kelompok yang sama dan dapat kita lihat bahwa
jumlahnya adalah 104.

Pewarnaan
Biasanya pewarnaan ini digunakan dalam pengubinan pengubinan. Di sini akan
disajikan satu contoh dan untuk memahami teknik lebih lanjut silakan mengerjakan soal
pada paket soal yang telah disediakan pada buku ini.

Nanang S

13
h

Contoh
Perhatikan gambar berikut ini

.
Gambar di atas disebut tetromino-T.
a. Buktikan bahwa papan catur berukuran 8  8 dapat ditutup dengan 16 tetromino-T.
b. Apakah papan catur berukuran 10 10 dapat ditutup dengan 25 tetromino-T?
(OSP SMA 2003)
Jawaban:
a. Perhatikan bahwa papan catur berukuran 8  8 dapat kita bagi menjadi empat
papan catur berukuran 4  4 , dan kita dapat menutup papan catur 4  4 ini dengan
4 tetromino-T seperti terlihat pada gambar di bawah ini.

Dari sini, tentu saja dapat kita simpulkan bahwa kita dapat menutup papan catur
8  8 dengan 16 tetromino-T.
b. Kita warnai papan catur 10 10 selang seling hitam putih, maka ada 50 petak
berwarna putih dan 50 petak berwarna hitam. Sekarang perhatikan bahwa setiap
tetromino-T dapat diwarnai dengan dua cara yaitu kita sebut dengan tipe 1
dan kita sebut dengan tipe 2. Andaikan papan catur 10 10 dapat kita tutup
dengan 25 tetromino-T, misalkan ada sejumlah x tipe 1 dan tentu saja 25  x tipe
2. Tipe 1 menutup 3 warna hitam dan 1 warna putih sehingga dengan x tipe 1
tertutup 3x warna hitam dan x warna putih. Sedangkan tipe 2, menutup 1 warna
hitam dan 3 warna putih sehingga dengan 25  x tipe 2 ini menutup 25  x warna
hitam dan 75  3x warna putih. Dengan demikian, yang tertutup adalah
3x  25  x  2 x  25 warna hitam dan x  75  3x  75  2 x warna putih. Karena
ada 50 warna hitam maka 2 x  25  50  x  12,5 (tidak mungkin karena x harus
merupakan bilangan bulat). Jadi kita tidak dapat menutup papan catur 10 10
dengan 25 tetromino-T.

Nanang S

14
DAFTAR PUSTAKA

Niven, I. ,Zuckerman, H.S., and Montgomery, H.L. 1991, An Introduction to the


Theory of Numbers, Fifth Edition, JohnWiley&Sons, Inc., NewYork,
Chichester, Brisbane, Toronto, Singapore.
Larson, L.C., 1983, Problem-Solving Through Problems, Springer-Verlag, New
York.
Coxeter, H.S.M., and S.L.Greitzer, 1967, Geometry Revisited, New Mathematical
Library 19, Mathematical Association of America.
Stanley, R.P., 1997, Enumerative Combinatorics, Cambridge University Press,
Cambridge.
Harmony South African Mathematics Olympiad 2011, South African Mathematics
Foundation, Pretoria.
UK Junior Mathematical Olympiad 2011, The United Kingdom Mathematics
Trust.
WA Junior Mathematics Olympiad, The University of Western Australia.
Netherlandse Wiskunde Olympiade, TU/e, Eindoven.
Olimpiad Matematik Kebangsaan, Malaysian Mathematical Sciences Society.
United State Junior Mathematical Olympiad, The Mathematical Association of
America.
Soal-soal OSN matematika SMP 2003-2011, kemendiknas.

35

View publication stats

Anda mungkin juga menyukai